You are on page 1of 123

B1.

1 Biochemistry and molecular biology

1) คนเป็ น DM ออกกำลังกำย ช่วยลดอะไร

A. Glycolysis
B. Insulin resistance
C. Glucagon
D. GLUT4 down regulate
E. Intestinal glucose absorption
2) A boy with smaller height. His mother recognizes that he often bangs
with things in the dark. Which substance is the most likely to be deficient
in this boy?
A. Retinol
B. Riboflavin
C. Cobalamin
D. Tocopherol
E. Ergoferol
3) ผูป้ ่ วยปวดบวมแดงที่ 1st MTP ของ Right foot ตรวจมี plasma uric acid 10 mg/dl และ
อำกำรจะหนักขึ้นเมื่อดื่ม whisky ถำมว่ำ alcohol ใน whisky จะทำให้เกิดกระบวนกำรใด

A. Increase purine uptake from liquor


B. Decrease salvage pathway of purine
C. Increase gastrointestinal uptake of uric acid
D. Decrease renal excretion of uric acid
E. Increase de novo synthesis of uric acid
4) ชำย 25 ปี เป็ น tendon xanthoma มี family history เป็ น premature coronary
artery disease. What would be high in this patient’s plasma?

A. VLDL
B. IDL
C. LDL
D. HDL
E. Chylomicron
5) คนไข้ high phenylalanine ไม่ควรกินอำหำรอะไรไปตลอดชีวติ

A. Caffeine
B. Aspartame
6) 8 week pregnant woman, อ้วกเยอะ malnutrition BP supine 110/70 upright
90/70 น่ำจะพบอะไรใน lab investigation

A. Na 149
B. K 6
C. HCO3 ??
D. pCO2 55 mmHg
E. ???
7) เล่นเทนนิสกลำงแจ้ง ผิวดำขึ้น โดยไม่มีอำกำรอื่น เนื่องจำก

A. Tyrosine
B. Dehydrocholesterol
8) ผูป้ ่ วย DM type II มี random glucose 350 plasma osmole 320 serum
ketone positive ถำมว่ำทำไม plasma osmole สูง

A. Glycolysis
B. Osmotic diuresis
C. Gluconeogenesis
D. Ketone production
E. ย่อย protein

9) A alcoholic cirrhosis patient with confusion is diagnosed hepatic


encephalopathy. Which of the Kreb's cycle intermediate is depleted in the
patient?
A. Malate
B. Fumarate
C. Oxaloacetate
D. Succinyl Coenzyme A
E. Alpha-ketoglutarate

10) ญ 60 ปี strict vegetarian; fatigue, moderate anemia น่ำจะขำดnutrientอะไร

A. Cobalamin
B. Pyridoxal
C. Copper
D. Lysine
E. Iron
B1.2 Biology of cells
1) Drug x inhibits telomerase activity and can be used in cancer. Which
phases does drug x interfere?
A. G0
B. G1
C. S
D. G2
E. M
2) Immotile cilia syndrome อวัยวะใดผิดปกติ

A. Alveoli
B. Fallopian tube
C. Small intestine
3) ผูป้ ่ วยอำยุ 40 ปี เป็ น follicular lymphoma พบว่ำมี Bcl-2 overexpression ถำมว่ำโปรตีนนี้
ไปรบกวนกระบวนกำรใด

A. Apoptosis
B. Angiogenesis
C. Differentiation
D. Cell adhesion
E. Cell cycle arrest
B1.3 Human development and genetics

1) Ectoderm ผิดปกติ ถำมว่ำอะไรยังปกติอยู่

A. Hair
B. Nail
C. Teeth
D. Keratinocyte
E. Langerhans cell
2) What is the mRNA transcripted from 5'TGGAGGCCA3' DNA?

A. 5'UGGCCUCCA3'
B. 5'ACCUCCGGU3'
3) เด็กพบ urine ไหลออกมำทำงสะดือ Which structure is the most likely cause?

A. Omphalocele
B. Gastroschisis
C. Umbilical hernia
D. Persistent vitelline duct
E. Persistent lumen of allantois
B1.4 Biology of tissue response to disease

1) เจ็บท้องร้ำวหลัง amylase ขึ้น ผ่ำพบ serosanguinous fluid white patch ที่ pancreas
ถำมว่ำ patch มำจำก
A. Coagulative necrosis
B. Liquefactive necrosis
C. White infarction
D. Saponification
2) ฉี ด collagen หน้ำเบี้ยว เพรำะเซลล์อะไร

A. Neutrophil
B. Macrophage
C. Eosinophil
3) COPD with edema both leg for 3 months PE : engorged neck vein,
hepatomegaly อะไรคือ physiologic change of liver

A. Edema
B. Infarction
C. Congestion
D. Fatty change
E. Bridging necrosis
B1.5 Gender, ethnic, and behavioral considerations affecting disease
treatment and prevention, including psychosocial, cultural, occupational,
and environmental
1) เป็ นเหมือน meningitis indian ink เจอวง halo which suggests cryptococcal
infection หมอส่งตรวจ anti-HIV ถำมว่ำ violate ethics ใด

A. Non-maleficence
B. Beneficence
C. Autonomy
D. Justice
E. Conflict of interest
2) ผูห้ ญิงเป็ น recurrent fungal vaginitis despite treatment มีสำมีทำงำนก่อสร้ำง มีลูก 2 คน
ถำมว่ำปั จจัยใดส่งผลให้ผหู ้ ญิงคนนี้เป็ นโรคบ่อยๆ

A. Malnutrition
B. Low socio-economic status
C. Poor hygiene
D.
E. ติดจำกสำมี

3) ทำงำนที่โรงงำน alloy urine test เจอ beta2-microglobulin เกิดจำกพิษของสำรใด

A. Pb
B. Cd
C. Hg
D. As
E. Mn
B1.6 Multisystem processes

1) A soldier heavy training body temp 40 hypotension fainting. Which is


the type of this symptom?
A. Heat syncope
B. Heat exhaustion
C. Classical heat stroke
D. Exertional heat stroke
2) ผญวิง่ 100m pH7.4-->7.2 Which process is response for the alteration of
the pH?
A. Breakdown of creatinine phosphate
B. Muscle glycogenolysis
C. Anaerobic glycolysis
D. Ketogenesis
E. Alpha oxidation
3) ญ 55 ปี น้ ำหนัก 80 kg สูง 155 และมีควำมดันโลหิ ตสูง มีควำมเสี่ ยงเป็ นโรคอะไรมำกสุด

A. Peptic ulcer
B. Diverticulosis
C. Osteoporosis
D. Endometrial cancer
E. Collagen vascular disease
B1.7 Principles of specimen collections and laboratory interpretations

1) คนไข้ DM มำ เจำะส่งน้ ำตำลในเลือดใส่สำรอะไรในหลอด

A. Sodium fluoride
B. EDTA
C. Sodium citrate
D. Heparin
2) Pap smear. Which solution is a specimen fixative for cytology?

A. 70% Alcohol
B. 95% Alcohol
C. 40% Formalin
D. 10% Formalin
E. 0.9% Sodium chloride

B1.8 Pharmacodynamic and pharmacokinetic processes

1) MRSA ให้ vancomycin ออกฤทธิ์ที่ใด

A. Cell wall
B. cell mem
C. ribosome
2) กินgentamycin แล้วต้องmonitorอะไร

A. AST, ALT
B. Cr, BUN
C. Ca2+, PO43-
D. FBS
E. CBC
3) กิน isosabide dinitrate คู่กบั sildenafil เกิด adverse reaction อะไร

A. Arrhythmia
B. Hypotension
4) กิน warfarin กับketoconazole แล้วbleedingเพรำะอะไร

5) ผูป้ ่ วยเป็ น HIV ได้รับยำ dideoxynucleoside drug ถำม mechanism ของยำ

A. Aberrant splicing
B. Viral gene mutation
C. Telomere synthesis
D. DNA chain terminator
E. Nucleotide degradation
6) ให้ยำ Glipizide modified release ควรแนะนำผูป้ ่ วยอย่ำงไรเกี่ยวกับกำรรับประทำนยำ
A. กินกับน้ ำอุ่น
B. ไม่ให้เคี้ยวหรื อหัก
C. ลดdose เองเมื่อเกิด hypoglycemia

7) เป็ นกรำฟ MIC ของยำชนิดหนึ่งที่มี Half-life 3 ชม. เมื่อให้ loading dose 100 mg IV it will
reach the therapeutic level at its peak level ถำมว่ำควรให้ยำยังไง

A. 800 mg OD
B. 200 mg ทุก 6 ชม
C. 300 mg ทุก 8 ชม
D. 400 mg ทุก12 ชม
E. 800 IV
8) ติดบุหรี่ ใช้ยำอะไรแก้ withdrawal symptom

A. Buproprion
B. Alprazolam
C. Amitryptiline
B1.9 Microbial biology and infection

1) A 50-year-old women has been presented with cirrhosis, high grade


fever, hemorrhagic bleb T40 BP 60/40 PR 130 After being punctated by
Spine of shell fish Hemoculture: Gram negative bacilli. Which one of the
following is the causative organism?
A. Vibrio vulnificus
B. Aeromonas hydrophilus
C. Klebsiella pneumoniae
D. Pseudomonas aeruginosa
E. Escherichia coli
2) A Japanese tourist patient came with acute abdominal intestinal
obstruction. He had a history of ingestion of raw marine fish ถำมว่ำปรสิ ตนี้คือ
อะไร

A. Angiostrongylus cantonensis
B. Anisakis simplex
C. Ascaris lumbricoides
D. Capillaria philippinensis
E. Trichuris trichiura
3) Generalized muscle pain, sea snorting, มี fang mark, Creatinine สูง,
most serious complication คือ

A. Hyperkalemia
B. Respiratory failure
C. Hypofibrinogenemia
D. Compartment syndrome
E. DIC
4) A 30-year-old man develops severe headache after ingestion of
uncooked land snails. Neck stiffness is noted on physical examination.
CSF examination reveals eosinophilia. Which of the following is the most
likely causative parasite?
A. Taenia solium
B. Trichinella spiralis
C. Capillaria philippinensis
D. Gnathostoma spirigerum
E. Angiostrongylus cantonensis
5) ผูห้ ญิงอำยุ55 เป็ นDM facial pain and headache พบ intra nasal black necrotic
อะไรซักอย่ำง คือหน้ำดำๆอ่ะ อะไรเป็ น causative organism
A. Rhizopus spp.

6) ผูห้ ญิงอำยุ20 มำขอคำแนะนำฉี ด vaccineป้ องกัน cancer ผลกำรตรวจเลือดพบว่ำ HBs Ag-, anti-
HBc+, anti-HBs+, anti-HAV Ig- ควรฉี ดvaccineอะไร

A. HBV
B. HAV
C. HCV
D. EBV
E. HPV
7) A 2-year-old fever rash with Koplik’s spot at buccal mucosal. how this
infection is transmitted?
A. Droplet
B. Feco-oral
C. Contact
D. Vertical
E. Vector-borne

8) A 50-year-old man anti-HIV positive man present with bilateral itchy at


groins region. The lesions show active broader and centrally clear
lesions. KOH preparation is shown below

What is the most likely causative agent?


A. Aspergillosis
B. Pityriasis rosea
C. Tinea cruris
9) ผูป้ ่ วย HIV-positive มำด้วยอำกำร diarrhea ตรวจ Stool examination พบ micro-
organism ขนำด 8-10 microns ดังรู ป ผูป้ ่ วยน่ำจะมีเชื้อจุลชีพใด
A. Isospora
B. Cyclospora
C. Microsporidia
D. Cryptosporidia
E. Mycobacterium avium complex
10) คนไข้ HIV มีไข้ ไอ หอบ 1 week, diffuse interstitial infiltration, sputum: cup-like
organism

A. Candida albicans
B. Penicillosis marneffei
C. Aspergillus ...
D. Pneumocystis jirovecii
E. Histoplasma capsulatum
B1.10 Immune responses

1) colon CA + metastasis มีอำกำร weight loss, malaise ถำมว่ำอำกำรเกิดจำกอะไร

A. TNF alpha
B. IL-10

2) ฉี ด tetanus toxoid ครบ ถ้ำติดเชื้อจะพบ immune แบบใด

A. IgG
3) A 25-year-old man presented with cough and dyspnea. He was
diagnosed lobar pneumonia. Sputum was collected for gram stain. Which
immune component is involved?

A. Cytokine
B. Eosinophil
C. Macrophage
D. Complement
E. Immunoglobulin
4) มี erythematous vesicular papule หลังจำกโดน beetle ต่อย ถำม immune
mechanism
A. Innate immunity
B. Anaphylactic reaction
C. Cell mediated immune response
D. Humoral immune response
5) กิน analgesic drug แล้ว 30 นำทีต่อมำเกิด generalized urticaria, face and lips
edema. What is the mechanism?

A. Neutrophil infiltration
B. CD 8+ T cell cytotoxic
C. Mast cell degranulation
D. Complement activation
E. ADCC
B1.11 Quantitative methods
1) โรค X ผูช้ ำยมีincidenceสูงกว่ำผูห้ ญิง แต่มีprevalenceเท่ำกันเพรำะอะไร

น่ำจะตอบเพรำะhigh case mortality rateในชำย

2) ยำ2อัน รักษำไมเกรนได้50%กะ70% ถำมข้อถูกผิดพวก relative risk

3) ทดลองครี มกันแดด ในคน40คน ครี ม A ทำบนแขนซ้ำย B บนแขนขวำ จับเวลำว่ำเมื่อไหร่ เกิด erythema


เปรี ยบเทียบระหว่ำง2ครี ม ใช้วธิ ีไหน

A. ANOVA
B. Chi-square
C. Paired t test
D. Independent t test
B2 Hematopoietic and Lymphoreticular Systems
1) เกิด blood loss ถำมว่ำสำรใดจะเพิ่มระดับขึ้นในเลือด

A. Ferritin
B. Transferrin
C. Hepcidin
D. Erythropoietin
E. Hemopexin
2) อะไรเป็ น initiator of clot lysis

A. Protein C
B. Protein S
C. Factor XIII
D. Fibrinokinase
E. Plasminogen activator
3) ผูป้ ่ วยanemia, dark urine Blood smear พบเซลล์เม็ดเลือดแดงดังรู ป ถำมว่ำควรตรวจอะไรต่อ
A. G6PD level
B. Hb test
C. OF test
4) Which one of the following is the indication of prenatal screening?

A. Hydrop fetalis
B. Wilson's disease
C. G6PD deficiency
D. Hemoglobinopathy
E. HS
5) An adult male patient with pancytopenia, plt 10,000 wbc 1500 hb 9 hct
27 ถำมว่ำควรส่ง lab investigation อะไรต่อไป

A. Bone marrow biopsy


B. Blood glucose
C. G6PD level
D. Folinic acid level
6) During blood transfusion a recipient has got fever and chill. What is the
main mechanism?
A. Cytotoxic T lymphocyte
B. IgE crosslink with RBC
C. Complement fixation
7) A 25-year-old man cc: abdominal pain and jaundice CBC: Hb 8 WBC
10,000 plt 100,000 (ไม่แน่ใจ) Blood smear: hypochromic microcytic,
anisocytosis 2+, spherocyte few, polychromasia few, basophilic stippling,
NRC 1/100 RBC Q: RBC มี defect ที่ไหน

A. Heme
B. Globin
C. Spectrin
D. G6PD
E. Glycophosphatidylinositol anchor protein
B3 Central and Peripheral Nervous Systems
1) A 30-year-old woman has frequent attack of throbbing and unilateral
headache, sensitive to light. What is the best medication for this patient?

A. Beta blocker
B. ACE inhibitor
C. Serotonin agonist
D. Nitric oxide donor
E. Calcium channel blocker
2) A man has been accidentally cut and his median nerve was severely
damaged. His doctor performs surgery for nerve reparation. Which cell
plays role in regeneration of nerve?
A. Neuron
B. Microglia
C. Fibroblast
D. Macrophage
E. Schwann cells
3) Which enzyme is abnormal in a patient with Parkinson disease?

A. Tyrosine hydroxylase
B. Dopamine hydroxylase
4) ผญ เป็ น depression +suicidal thinking ถำมว่ำสำรสื่ อประสำทใดผิดปกติ

A. Serotonin
B. Dopamine
C. Acetylcholine
D. Glutamate
E. GABA
5) สมองมี lesion ดังรู ป (ที่เป็ นสี แดงสด) ถำมว่ำอำกำรจะเป็ นอย่ำงไร

A. Ipsilateral paresis เป็ นมำกที่ leg


B. Ipsilateral paresis เป็ นมำกที่ arm
C. Contralateral paresis เป็ นมำกที่ leg
D. Contralateral paresis เป็ นมำกที่ arm
E. Contralateral paresis เป็ นมำกที่ head

6) ญ progressive muscle weakness เจอ dysphagia ptosis when asked to


perform repeated blinking ถำมว่ำมี Antibody ต่ออะไร

A.
B.
C.
D.
E. Anti-acetylcholine receptor

B4 Skin and Related Connective Tissue


1) A 30-year-old man has painful skin and oral mucosal blister. Skin
biopsy reveals suprabasal separation of keratinocyte. Which cell
adhesion molecule is defective in this patient?
A. Desmosome
B. Zonula adherens
C. Hemidesmosome
D. Gap junction
E. Macula adherens
2) sunburn แดง แสบ ไม่มี blister ถำมว่ำ lesion ที่ผวิ หนังชั้นใด

A. Epidermis
B. Papillary Dermis
C. Reticular dermis
D. Hypodermis
E. Subcutaneous tissue
3) ผูป้ ่ วยมี stiffed finger skin, Tense neck, cleavage furrow รอบปำก มี collagen
deposit ขนำนกับ something ถำมว่ำน่ำจะพบอำกำรอะไรร่ วม

A. Dysphagia
B. Acne
C. Urolithiasis
D. Oliguria
4) คัน มี vesicle บน erythematous base มี รอย scratch ตำม fingers, toes, buttock
น่ำจะเป็ นอะไร

A. Scabiasis
5) A 2-year-old girl presents with facial dermatosis. Topical corticosteroid
is prescribed. Which of the following is the most appropriate drug for this
patient?
A. Clobetasone
B. Prednisolone
C. Triamcinolone
D. Betamethasone
E. Hydrocortisone
6) ให้อำกำร SLE มำ ถำม specific diagnostic test

A. Anti-dsDNA

7) Harlequin ichthyosis มี defect ที่ใด

A. Keratinocyte
B. Adipocyte
C. Lanugo hair
D. Sebaceous gland
8) เด็ก 10 ขวบมำด้วย purpura ก่อนหน้ำนี้เคยกินยำ penicillin เพื่อรักษำโรคติดเชื้อ streptococcus
ต่อมำไตผิดปกติ ส่งตรวจไต ไม่พบ immune deposit แต่พบ antibody to myeloperoxidase
ถำมว่ำถ้ำทำ skin biopsy จะพบอะไร

A. Leukocytoclastic vasculitis
B5 Musculoskeletal System
1 A 20-year-old man has right ankle sprain. PE: tenderness at medial
malleolus. Which of the following structures is most likely to be injured?

A. Deltoid ligament
B. Talonavicular ligament
C. Tibialis anterior tendon
D. Calcaneofibular ligament
E. Calcaneonavicular ligament
2) ชำยอำยุ 50 ปี เป็ น acute gout attack ได้กินยำ colchicine ต้องระวังภำวะอะไร

A. Rash
B. Nausea
C. Vomiting
D. Dizziness
E. Diarrhea
3) ผูป้ ่ วยโรคเก๊ำท์กินอะไรปลอดภัยที่สุด

A. Egg
B. Beef
C. Salmon
D. Asparagus
E. Bean sprout
4) A 70-year-old woman with pain in both knees for 5 years. X-ray finding
of both knee shows osteophyte formation. Which of the following
substance is most likely to be reduced regarding to this condition.

A. Filamin
B. Laminin
C. Fibronectin
D. chondroitin sulfate
E. MMP
5) เด็กชำยอำยุ 5 ปี มี fever, joint pain, diffuse ... rashes at 2 weeks after having a
sore throat. พบ several erythematous rash, increase ESR.

A. c-ANCA titer
B. Anti-dsDNA titer
C. Anti-DNAse B titer
D. Throat swab culture
E. Hemoculture
6) เป็ น OA ได้ยำ bisphosphonate ถำมว่ำมี mechanism อะไร

A. Increased
B. Increased osteocyte
C. Increased osteogenic stem cell
D. Decreased osteoclast activity
E. Decreased apoptosis of osteoblast
7) หลังทำกำรผ่ำตัด hip replacement พบ Trendelenburg gait ถำมว่ำไปทำลำยโดน nerve
อะไร

A. Sciatic nerve
B. Femoral nerve
C. Saphenous nerve
D. Tibial nerve
E. Superior gluteal nerve
8) ผูป้ ่ วยมำด้วยอำกำร Acute onset pain, swell, 1 วัน เจำะ synovial fluid ต้องดูอะไรเป็ นอย่ำง
แรกเพื่อ diagnosis

A. Mucin clot test


B. Glucose determinant
C. White count
D. Air dry smear for gram
E. Wet preparation for microscope
9) ผูช้ ำยอำยุ40ปี มำด้วย Right MTP joint บวม uric acid=10 อำกำรจะแย่ลงถ้ำกินเหล้ำ เพรำะอะไร

A. Increase uptake of purine from the liquor


B. ลด salvage pathway ของ purine
C. เพิ่ม GI absorption ของ uric acid
D. ลด renal reabsorb ของ uric acid
E. เพิม่ de novo ของ uric acid

10) ผูป้ ่ วยปวดบวมแดงที่ 1st MTP ของ Rt. foot ตรวจมี plasma uric acid 10 mg/dl และอำกำร
จะหนักขึ้นเมื่อดื่ม whisky ถำมว่ำ alcohol ใน whisky จะทำให้เกิดกระบวนกำรใด
A. increase purine uptake from liquor
B. decrease salvage pathway of purine
C. increase GI uptake of uric acid
D. decrease renal excretion of uric acid
E. increase de novo synthesis of uric acid
B6 Respiratory System
1) ชำย อำยุ 30 ปี chronic productive cough, infertility, recurrent pulmonary
infection, CT scan = sinus inversus อำกำรพวกนี้เกิดจำก ควำมผิดปกติที่ organelle อะไร

A. Mitochondria
B. Microtubule
C. Lysosome
D. Peroxisome
2) เด็ก tantrum กลั้นหำยใจแปปนึง ก็ตอ้ งหำยใจใหม่ เพรำะอะไร

A. Paco2
B. Pao2
3) ฝำแฝด ชื่อ A กับ B มี oxygen consumption = 15 และ 25 ตำมลำดับที่ HR 120 เท่ำกัน ถำมว่ำอะไร
ที่ B มีมำกกว่ำ A

A. Maximal Heart rate


B. Muscle mitochondria density
C. Tissue oxygen extraction ability
D. Stroke volume
E. Sympathetic tone
4) A child has come to the doctor with barking cough and stridor without
hoarseness. Which structure is affected?

A. Larynx
B. Trachea
C. Bronchus
D. Bronchiole
E. Alveoli
5) A man was founded unconscious and near drowning. What is the most
probable lung change?
A. Decrease surfactant
B. Decrease elastic recoil
C. เพิ่ม alveolar dead space

6) Premature child มี cyanosis, tachycardia, subcostal retraction. Which


cell is abnormal in this child?
A. Alveolar cell
B. Endothelial cell
C. Alveolar macrophage
7) ผูป้ ่ วยเด็ก 1 ปี เป็ นไข้สูง 2 วัน กินยำลดไข้แล้ว hyperventilation pH 7.3, PCO2 20, PO2 95,
HCO3 12 จงอ่ำน acid-base

A. Respiratory acidosis
B. Metabolic acidosis
C. Compensated metabolic acidosis
D. Mixed respiratory and metabolic acidosis
E. Mixed metabolic acidosis with respiratory alkalosis
8) A patient is presented with orthopnea, dyspnea, pink-frothy sputum,
pitting edema both legs, cardiomegaly. His chest x-ray reveals patchy
effusion. Which is the most likely diagnosis?

A. Acute pulmonary edema


B. Pleural effusion
C. Adult respiratory distress syndrome.

9) อำยุ 70 ปี เป็ น dyspnea, fine crepitation, ...... อะไรเป็ น histopathology ของรำยนี้

A. Intra-alveolar floccular pink…


B. Extravasated ... รอบๆ capillary

B7 Cardiovascular System
1) BP 100/60, HR 120/min
ESV=160, EDV=200 หำ ejection fraction เป็ น %

A. 20 %

2) Melena, Hematemesis แล้ว hypovolemic shock ถำมว่ำ cardiovascular จะ


เปลี่ยนแปลงยังไง

A. Negative chronotropic
B. Decrease contraction
C. Decrease capillary hydrostatic pressure
D. Increase preload
3) Hypertension, angina อะไรช่วยผูป้ ่ วยคนนี้

A. Increase cardiac contractility


B. Decrease cardiac conductivity
C. Dilate vein and artery
4) อำยุ 70 ปี เป็ น angina, dyspnea กลไกอะไรจะ improve อำกำร

A. Decrease conductivity
B. Decrease ...
C. Artery and venous dilation
D. Increase preload
E. Increase contractility
5) ชำย 70 ปี เป็ น chronic kidney disease (ให้ Na+, Cl-, K+ มำ-> Kสูง) ถำมว่ำ ECG จะเป็ นไง
A. wide P
B. ST suppression
C. Tall peak T-wave
D. prolong QT
E. prolong PR
6) ชำยอำยุ 60 มีอำกำรเจ็บหน้ำอกเมื่อออกแรง อำกำรดีข้ ึนเมื่อพัก กลไกของยำที่ชวั่ ลดอำกำรดังกล่ำวคืออะไร

A. Lipoxygenase inhibition
B. Phospholipase A2 inhibition
C. Nitric oxide production activation
D. Na+/K+ ATPase activation

7) Patient with stable angina: Which drug mechanism would relieve the
pain?
A. Inhibit LO
B. Inhibit PhospholipaseA2
C. Inhibit K channel
D. NO synthesis
E. Decrease Na-K pump activity

8) เจ็บหน้ำอก ร้ำวไปไหล่และแขนซ้ำย มีเหงื่อออก มำ 1 ชม จะมีผลต่อช่วงไหนของ ECG มำกที่สุด?

A. P wave
B. PR interval
C. QRS complex
D. QT interval
E. T wave
9) A heavy smoker man with Pain in calves when have a short distance
walking. His finger and toes are cold and pale. Which of the following is
the most clinical diagnosis?
A. Giant cell arteritis
B. Takayasu arteritis
C. Thromboangiitis Obliterans
10) A patient was presented with heart failure and hepatomegaly. What is
the most likely histopathological finding of liver ?
A. Edema
B. Infarction
C. Congestion
D. Fatty change
E. Bridging necrosis
11) A 55-year-old man has acute pain at substernal region. The pain
radiate down ulnar side of left forearm. CK-MB = 25 ng/ml (5-10 ng/ml).
What will exist in 48 hours?
A. Myelin figure
B. Neutrophil infiltration
C. Fibroblast proliferation
D. Epithelium proliferation
E. Hemosiderin-laden macrophage

12) ฟังที่ 5th intercostal space ตัดกับ midclavicular line. ฟังอะไร?

A. Mitral valve
B. Papillary muscle
C. Tricuspid valve
D. Aortic valve
E. Pulmonary valve
13) ให้ EKG มำ พร้อมบอกว่ำ เสี ยงหัวใจฟัง distant

A. Cardiac tamponade
B8 Gastrointestinal System
1) หญิงกินอำหำรโปรตีนสูง คำร์โบไฮเดรตต่ำเพื่อไดเอท ฮอร์โมนอะไรจะเพิ่มขึ้น
A. Cortisol
B. Secretin
C. Glucagon
D. Epinephrine
E. CCK
2) Car accident โดน blunt trauma เข้ำผ่ำตัดใหญ่ หลังผ่ำไม่กี่วนั เจ็บ epigastrium และพบ
coffee-ground ใน NG tube ทำendoscopeพบแผลตื้น ๆ ทัว่ กระเพำะ Gastric biopsy พบ
neutrophil เยอะมำก, erosion, new hemorrhage. ถำมว่ำเป็ นโรคอะไร

A. Peptic ulcer
B. Curling ulcer
C. Acute gastritis
D. Reactive gastritis
3) ญ 70 ปี present with anorexia, vomiting, abdominal pain, weight loss
6kg in 2mo. PE: cachexia, non-tender cervical lymphadenopathy,
abdominal CT reveals shrunken, diffusely thicken wall of stomach. Which
histological will be found in this patient?
A. Granulomatous inflammation
B. Keratin pearl and intercellular bridge
C. Diffuse fibrosis
D. Infiltrate of signet-ring cell
E. Well-formed gland

4) Sudden severe abdominal pain at mid-epigastric region with diffuse


tenderness in all quadrant. Enzyme Amylase and lipase ขึ้น. ถำมว่ำ กำรพบอะไรจะ
ทำให้ prognosis แย่สุด

A. Urea breath test


B. Hypocalcemia
C. Hypercholesterolemia
5) Patient complained dull, ill-define pain sensation in her umbilical region
and her physician suspected inflammation in her abdomen. Which of the
following is the most likely site of inflammation?
A. Liver
B. Stomach
C. Gall bladder
D. Small intestine
E. Parietal peritoneum
6) ผูป้ ่ วยชำยอำยุ 50 ปี มำด้วย jaundice 3 เดือน มีประวัติดื่มสุรำอย่ำงหนักมำ 20 ปี อะไรบอกว่ำเป็ น portal
hypertension
A. Jaundice
B. Ankle edema
C. Coagulopathy
D. Gynecomastia
E. Esophageal varices
7) ผูป้ ่ วยชำยอำยุ 55 ปี เป็ น alcoholic cirrhosis เกิด portal-systemic encephalopathy
ควรใช้ยำอะไรลด enteric toxic ของผูป้ ่ วย

A. Lactulose
B. Ranitidine
C. Sucralfate
D. Loperamide
E. Omeprazole
8) Gall bladder thicken wall. พบนิ่ว 10 ก้อน เป็ น greenish-brown multifacet
stones ขนำด 0.5-1 cm ถำมว่ำจะพบอะไรร่ วม

A. Hypercholesterolemia
B. อะไรสักอย่ำงเกี่ยวกับ Calcium หรื อกระดูก

9) ผูช้ ำยอำยุ45 น้ ำหนักลด มีdysphagia พบก้อนที่ lower esophagus เป็ น squamous cell
CA ถำมว่ำassociateกับอะไร

A. Coffee drinking หรื อ Alcohol consumption ไม่แน่ใจ


B. Vegetarian consume
C. Smoking
D. Barrett's esophagus
E. Helicobacter infection
10) A 60-year-old woman has blood leakage from intestinal hemorrhage.
Where is the most likely location of blood accumulation in quantity when
in supine, bedridden position?
A. Hepatorenal pouch
B. Rectouterine pouch
C. Rectovesical pouch
D. Vesicouterine pouch
E. Hepatoduodenal pouch
11) A male patient with drowsiness and confusion. PE: jaundice, spider
nevi, ascites. Lab: total protein 4.5, albumin 1.8, total bilirubin 14.8, direct
bilirubin 10.4, AST 130, ALT 28, ALP 55, ammonia 280. What is the
cause of his clinical manifestations?
A. Alcoholism
B. Fulminant hepatitis
C. Hepatic encephalopathy
D. Wernicke encephalopathy
12) Omeprazole ออกฤทธิ์ที่เซลล์ใด

A. parietal cell
13) คนไข้เป็ น achalasia nerve อะไรผิดปกติ

A. celiac n.
B. vagal n.
C. phrenic n.
D. myenteric plexus of esophagus
E. submucosal plexus of esophagus
14) Rice watery stool มีmechanism ยังไง

A. cAMP
B. tyrosine kinase
15) กินยำ แก้ inflammatory bowel disease มีอำกำร leukopenia,
thrombocytopenia, high MCV RBC, alopecia, mucositis กินอะไรแก้ได้

A. Folic acid
B. Vitamin B12
C. Vitamin B6
D. C-GSF
E. Leucovorin
B9 Urinary System
1) pH 7.33, PCO2 70, HCO3 36
Which one of the following is physiological compensation?
A. Hyperventilation
B. Excretion of NH4
C. Secretion of HCO3
D. Decreased titratable acid secretion
E. Decreased NaCl reabsorption
2) A 10 year-old has headache, puffy eyelid and bilateral leg edema for 2
days. Two week ago, she had sore throat and fever relieved by
consuming over-the-counter drug. PE: BP 140/80. UA: protein 2+, RBC 5-
10/hpf, red cell cast 1-2/lpf. Which of the following condition is most
compatible with the clinical presentation?
A. Lupus nephritis
B. Acute kidney injury
C. Minimal change glomerulonephritis
D. Drug-induced IN
E. Acute poststreptococcal glomerulonephritis
3) A 6-year-old boy is presented with proteinuria 4+,
hypercholesterolemia, and hypoalbuminemia. What is the most likely
finding when renal biopsy is performed?
A. Subendothelial immune deposit
B. Diffuse foot process effacement
C. Subepithelial hump like deposit
4) พบ dysmorphic RBC, RBC 5-10/field, มี massive hematuria ถำมว่ำเกิดอะไรก่อน
หน้ำนี้

A. Penicillin hypersensitivity
B. Bacterial skin infection 3-4 weekก่อน
C. Bacterial pharyngitis 1-2 วันก่อน
D. URI 1-2 week ก่อน

5) กินกล้วยเยอะ มีโพแทสเซียมสูง ถำมว่ำกลไกใดทำให้โพแทสเซียมในเลือดยังมีค่ำปกติ

A. ขับ K+ ออกที่thick ascending limb


B. เพิม่ Na+-K+ ATPase pump ที่ collecting duct

6) มีอำกำรหูหนวก & อำกำรทำงไต ถำมว่ำ เสี ย collagen type ใด

A. 1
B. 2
C. 3
D. 4
E. ทุก type

7) After eating high protein diet for 4 hr. Which mechanism causes high
urine output?
A. Increased plasma protein
B. Increased plasma tonicity
C. Increased plasma sodium
D. Decreased urine protein
E. Decreased sodium reabsorption
B10 Reproductive System
1) ผูห้ ญิง dysmenorrhea ให้ฮอร์โมนเพื่อบรรเทำอำกำร ถำมว่ำเป็ นฮอร์โมนอะไร
A. Progesterone
B. Cortisol
C. Estrogen
D. Testosterone
E. DHT
2) Straddle injury from motorcycle accident พบ inferior pubic ramus
fracture ถำมว่ำโดน artery อะไร

A. Inferior rectal artery


B. Posterior scrotal artery
C. Internal pudendal artery
D. Deep artery of the penis
E. Obturator artery
3) Internal penile injury พบ swelling shaft of penis แต่ no swelling of
scrotum, และไม่มีurine extravasation ที่ perineum and anterior abdominal
wall. Which fascia hold extravasated urine?

A. Colles fascia
B. Buck fascia
C. Scarpa fascia
D. Dartos fascia
E. Camper fascia
4) Difficult start urination and drip ตอนเสร็ จ digital rectal: soft enlarge... PSA 3
มั้ง ถำมยำยับยั้งอะไร

A. 5-α reductase
B. Aromatase
C. 17-hydroxylase
D. 11-..

5) ผูห้ ญิงอำยุ 35 ปี amenorrhea มำ 2 เดือน แต่ก่อนประจำเดือนมำปกติเสมอ ซักประวัติ Last


menstrual period วันที่ January 10th, 2015 ถำมว่ำจะเกิดกำร Transplantation เมื่อไหร่

A. January 17th, 2015


B. January 24th, 2015
C. January 31st, 2015
D. February 7th, 2015
E. February 14th, 2015
6) กำรทำ pudendal nerve block ก่อนทำคลอด ใช้ landmark of injection ที่ใด

A. Ischial spine
B. Ischial ramus
C. Ischial tuberosity
D. Obturator foramen
7) A female patient has a chief complaint of uterine prolapse. Which
structure is weakened in this patient?
A. Perineal body
B. Round ligament of uterus
C. Broad ligament of uterus
D. Transverse perineal muscle
E. Transverse cervical ligament
8) A 50-year-old menopause women with hot flush ได้ EE (Ethinyl estradiol)
0.01 mg/day. What is potential risk?
A. Hypotension
B. Liver cancer
C. Breast cancer
D. Ovarian cancer
E. Coag factor 5,7,9,10 defect
9) A 35-year-old patient มีอำกำร hypermenorrhea dysmenorrhea ได้รับกำรวินิจฉัยว่ำ
เป็ น endometriosis ได้รับ danazol ถำมกลไกยำ

A. Androgen receptor stimulation


B. Glucocorticoid receptor stimulation
C. Estrogen receptor stimulation
D. Aromatase inhibitor
E. 5-α reductase inhibitor

10) A 25-year-old woman with lactational amenorrhea is breastfeeding for


6 months. Which of the following is underlying of this condition?

A. Stress-induced anovulation
B. Decreased GnRH secretion
C. Decreased oxytocin secretion
D. Decreased prolactin secretion
E. Decreased dopamine secretion
11) ช 45 ปี พบ reducible mass ที่ขำหนีบ วินิจฉัยว่ำเป็ น direct inguinal hernia ถำมว่ำ
reducible mass อยู่ medial ต่ออะไร

A. Lateral umbilical fold


B. Rectus abdominis
C. Spermatic cord
D. Inguinal ligament
12) A 30 year-old pregnant women attends an antenatal care clinic. A
fetal detect is suspected. Which is the indication for a prenatal
diagnosis?
A. Hydrops fetalis
B. Wilson disease
C. G-6-PD deficiency
D. Hemoglobinopathy
E. Hereditary spherocytosis

13) Which of the following structure has


defect in development?
A. Urogenital membrane
B. Cloacal membrane
C. Urorectal membrane
D. Urogenital fold
E. Genital tubercle
B11 Endocrine System
1) เป็ น SLE กิน prednisolone มีอำกำร muscle weakness ถำมelectrolyte

A. Hypokalemia
2) Thyroid gland biopsy from patient reveals psammoma bodies with
nuclear groove ถำมว่ำโรคอะไร

A. Papillary CA
B. Follicular CA
C. Follicular adenoma
3) Which mechanism in thyroid hormone metabolism is primarily affected
by Lugol solution?
A. Coupling
B. Secretion
C. Deiodination
D. Iodine uptake
E. Organification
4) T3 T4 ปกติ TSH สูง มีกอ้ นหน้ำคอ What is the most likely diagnosis

A. Multinodular goiter
B. Secondary hyperthyroidism
C. Secondary hypothyroidism
D. Subclinical hyperthyroidism
E. Subclinical hypothyroidism
5) ผูช้ ำยอำยุ70เป็ นDM มีอำกำร ไข้ ไอ หอบ ผลแลป: plasma Osmolarity 320, blood
Glucose 350, Renal ketone bodies positive ถำมว่ำOsmolarity สูงเพรำะอะไร
A. Glycolysis
B. Osmotic diuresis
C. Gluconeogenesis
D. Ketone body synthesis
E. Protein breakdown
6) ผูป้ ่ วย DM type II มี random glucose 350 plasma osmole 320 serum
ketone positive ถำมว่ำทำไม plasma osmole สูง

A. Glycolysis
B. Osmotic diuresis
C. Gluconeogenesis
D. Ketone production
E. ย่อย protein
7) เด็กอำยุ 15 ปี เป็ น DM มำด้วยอำกำร polyuria, polydipsia, dyspnea ตรวจร่ ำงกำยพบ BP
80/50 P 120/min R 28/min blood glucose 400 mg/dL ควรทำอย่ำงไรเป็ นขั้นตอนแรก?

A. Dopamine IV injection
B. Dobutamine IV injection
C. Dextran solution IV infusion
D. Mannitol solution IV infusion
E. Normal saline solution IV infusion
8) A 50 year-old has parathyroid tumor. Lab analysis reveals calcium 11
and phosphate 3 mg/dL. Which of the following mechanism is
responsible for this blood result?
A. Decrease bone resorption
B. Decrease renal calcium reabsorption
C. Decrease intestinal calcium absorption
D. Increase active vitamin D
E. Increase renal phosphate reabsorption
9) คนเป็ น truncal obesity, striae ... ต้องตรวจแลปอะไรเพิ่ม

A. VMA
B. Dextromethorphan test
C. Dexamethasone suppression test
10) ทำไมเวลำตื่นนอนปั สสำวะถึงออกมำน้อย แม้วำ่ ก่อนนอนจะกินน้ ำไป 1.5 ลิตร

A. Increased ADH
B. Increased Epinephrine
C. Increased Renin
D. Decreased Angiotensin II
E. Decreased Aldosterone

B1.1. Biochem & Molecular Genetics

1. ตึกถล่ม ติดอยูใ่ นตึก 3 wk ใช้กลไกใดสร้ำงพลังงำน : B-oxidation


2. หำ mRNA จำก template DNA (เหมือนข้อสอบเก่ำ)
3. วิง่ 100m ทำให้ pH ลดลงจำก 7.4 เหลือ 7.25 เกิดจำกกลไกใด : anaerobic glycolysis
4. Hepatic encephalopathy which intermediate in TCA cycle is
decreased? (ข้อสอบเก่ำ)
a. OAA
b. Alpha ketoglutarate
c. Malate
d. succinyl-CoA
5. พบ เม็ดเลือดขำวมี Eosinophilic inclusion ในbone marrow วินิจฉัย lysosomal
storage disease มีควำมเกี่ยวข้องกับสำรใด(Gaucher disease)
a.glycogen

b.glycolipid

C.sphingolipid

d.phospholipid

e.glycoprotein

B1.2. Biology of cell

1. ประวัติ recurrent respiratory infection บ่อยๆ ตรวจพบcilia dysfunction ->


อวัยวะใดมีโอกำสเสี ยด้วย : fallopian tube, small intestine
2. telomerase inhibitor ยับยั้งที่ระยะใดของ cell cycle

1. G0
2. G1
3. S
4. G2
5. M
3. recurrent respiratory tract infection, infertility อะไรผิดปกติ (บ่ำย)

1. mitochondria
2. microtubule

B1.3. Human development and genetics


1.
Pedigree นี้มีกำรถ่ำยทอดแบบใด :AD, AR, XD, XR

2. ถำมหำโอกำสเป็ น carrier ในประชำกำรปกติที่มี prevalence ของโรค AR = 1/40000

1. 1/100 ,2. 1/200

3. Indication for prenatal diagnosis : hydrop fetalis, wilson disease,


G6PD def, hemoglobinopathy
B.1.4. Normal immune response

1. ติดไวรัส what is the first cell to respond (ช้อยส์ไม่มี NK cell)


a. Basophils
b. Eosinophils
c. Neutrophils
d. macrophage
e. lymphocyte(ตอบ เพรำะ NK Cell is a type of lymphocyte)
2. เด็กอำยุ10 ปี ปวดหัว หอบเหนื่อย ไข้ เจ็บหน้ำอก ย้อมเสมหะพบ gram positive diplococci
lancet shape กลไกที่สำคัญที่สุดในกำรกำจัดเชื้อนี้คือ
a. Opsonization
b. ADCC
c. TLR4 activation
d. Cytotoxic T cell
e. Superantigen recognition
3. เด็กติด S. aureus บ่อยๆ ผล NADPH oxidase ผิดปกติ กลไกลใดในกำรกำจัดเชื้อมี defect
a. Phagocytosis
b. chemotaxis
4. เด็กติดเชื้อง่ำย ไม่พบ thymus gland เด็กคนนี้จะไม่พบอะไรอีก

1. thyroid gland
2. adrenal gland
3. parathyroid gland(Ans)
5. beetle กัดเป็ น blister เป็ นรอยแดง เกี่ยวข้องกับอะไร

1. humoral immunity
2. cell mediated immunity
6. A 12 year-old girl with asthma and allergic to dust mite has been
treated with desensitized therapy. What is the mechanism of
desensitized therapy? (ข้อสอบเก่ำ)

1. increase IgA production


2. increase IgG production(Ans)
3. decrease IgE production
4. decrease mast cell secretion
5. decrease cytotoxic T cell activation
B.1.5 Pathogenesis , Pathophysiology , Basic Pathologic process , LAB
interpretation
1. ตรวจ Glucose ใส่ อะไร → NaF(Ans if store more than 2 hrs), sodium
citrate, heparin, plain tube, EDTA
2. pap smear(ไม่ได้บอกตรงๆ) screening for cervical cancer ใส่ อะไรน? -> 95
ethanol(Ans), 70% ethanol, normal saline, formalin
3. จะส่ง CSF ต้องเก็บไว้ ที่ อุณหภูมิไหน
1.-25

2. -4

3.4

4 25
5. 37

4. CBC จะ เลือก tube ไหน

1. NaF tube
2. EDTA tube(Ans)
3. Plain tube
4. Heparin tube
5. แม่มำตรวจ rubella ส่ง anti-Rubella IgM แต่ส่งตรวจไม่ได้ทนั ที จะทำอย่ำงไร
1. serum -20C
2. serum room temp
3. plasma 4 C
4. plasma room temp
5. Whole blood 4 C
6. ผูป้ ่ วยชำยอำยุ 50 ปี เป็ น chronic hepatitis ทำ liver biopsy พบ fibrotic change พยำธิ
สภำพดังกล่ำวเกิดจำกเซลล์ใด

1. Hepatocyte
2. Kupffer cell
3. Hepatic stellate cell(Ans aka Ito cell or perisinusoidal cell, major
cell type involved in liver fibrosis)
4. Intrahepatic lymphocyte
5. Sinusoid endothelial cell

B.1.6. Gender,ethics , behavioral consideratio affecting disease treatment


, and prevention including psychosocial cultural and environment
1. หมอตรวจเจอ cryptococcal meningitis แล้ว ตรวจHIV คนไข้เองเลย -->ผิดอะไร-->ผิด
autonomy
2. ผูป้ ่ วยเป็ น HT คุมควำมดันดี อยำกขอไปรับรองแพทย์ไป early retire ถำมว่ำจะให้ไหม+เพรำะอะไร
1.yes, autonomy

2. yes, physician duty

3. no, non-ethical conduct

4 no, injustice
3. หมอบอกว่ำเป็ นมะเร็ ง แล้วผป.ไม่เชื่อ ผูป้ ่ วยไม่สนใจทำเป็ นไม่ได้ยนิ

1. denial
2. regression
3. Suppression
4. Acting out
4. ให้น้ ำหนัก ส่วนสูงของหญิงคนหนึ่ง มำ

1. underweight
2. Normal
3. Obesity grade 1
4. Obesity grade 2
5.

B1.7. Multisystem process

1. ทหำรใหม่ถูกให้ฝึกกลำงแดด อุณหภูมิร่ำงกำย 40 องศำ + อำกำรต่ำงๆ ถำมว่ำ เป็ นอะไร


1. heat syncope

2. heat exhaustion

3. classical heat stroke

4. exertional heat stroke

5. cerebrovascular stroke

2. ผูป้ ่ วยหญิง ตั้งครรภ์ได้ 38 wk อยำกให้เด็ก เกิด วันนี้ เนื่องจำกเป็ น ศิริมงคล หมอก็ยนิ ยอม ทำ
cesarean section ให้ หมอ ผิดจรรยำบรรณข้อไหน
1 beneficence
2. Justice

3. Non maleficence

4. autonomy

3. pH 7.33, Pco2 70, HCO3- 36 ถำมว่ำผูป้ ่ วยมีกำร compensation ด้วยกลไกได้

1. Hyperventilation
2. Decreased NH4+ excretion
3. Decreased HCO3- secretion
4. Increased …
5. Increased NaCl reabsorption

B1.8 General Pharmacology

1. Antihypertensive drug, ankle edema and flushing มี Cr 3.0 mg/dl


a. Enalapril
b. Amlodipine(Ans เพรำะยำนี้ทำให้เกิด ankle edema และใช้ได้ในคนที่มี Cr สูง)
c. losartan
2. Angina when exertion not at rest, mechanism of drug

1. Increase NO production
3. 60 kg, Vd 0.5 L/kg, want concentration 100 mg/L, oral bioavailability
30%, what is loading dose?

1. 100 mg
2. 300 mg
3. 1000 mg(Ans 0.5x60x100/0.3)
4. antimycobacterial orange urine and sweat

1. rifampin(Ans)
2. ofloxacin
3. isoniazid
4. ethambutol
5. pyrazinamide
5. target of vancomycin

1. Cell wall(Ans bind to D-Ala-D-Ala)


2. Cell membrane
3. Topoisomerase II
4. RNA polymerase
6. ให้กรำฟ MIC มำบอกว่ำยำมี half life 3 ชม, ถ้ำให้ 100mg ยำจะมีควำมเข้มข้นสูงสุดอยูใ่ นค่ำ MIC ควร
บริ หำรยำอย่ำงไร

1. 800 mg once daily


2. 200 mg ทุก 6 ชม
3. 300 mg ทุก 8 ชม
4. 400 mg ทุก 12 ชม
5. 800 mg ให้ต่อเนื่อง IV continuous
7. ยำชนิดหนึ่งมี hepatic clearance 80% หำก GFR ลดลง 50% ต้องลด maintenance dose
ที่จะให้เหลือร้อยละเท่ำใด

1. 40
2. 50
3. 60(Ans)
4. 80
8. กินยำฆ่ำแมลง แล้วกินatropine เหลืออำกำรอะไรอยู่ -> muscle fasciculation(Ans เพรำะ
Atropine ยับยั้งแค่ Muscarinic receptor)

9. Normal subject ก่อนให้ยำ BP 120/80 HR 70 หลังให้ยำ BP 130/60 HR 90 ยำกระตุน้


receptorใด

a. alpha 1
b. alpha 2
c. beta1
d. alpha1, beta2
e. alpha 1, beta 1
10. A patient on warfarin. She has bleeding per gum while brushing her
teeth. She is currently on ketoconazole. ยำสองตัวนี้สมั พันธ์กนั อย่ำงไร

1. decrease hepatic biotransformation


2. Decrease renal excretion
3. แย่งจับกับ albumin แบบ Competitive
4. Increase volume of distribution
5. Decrease (หรื อ increase นี่แหละ ==”) interstitial อะไรซักอย่ำง
11. ให้ยำชนิดหนึ่ง narrow therapeutic index ยำนี้มี half life 24 hr. ควรเจำะดู plasma
drug concentration เมื่อให้ยำไปแล้วกี่ชวั่ โมงเพื่อจะให้ maintenance dose

1. 12
2. 24
3. 48
4. 72
5. 96
12. ผูป้ ่ วยหญิง 30 ปี มีอำกำรเจ็บคอ ก่อนหน้ำนี้ได้รับกำรวินิจฉัยว่ำเป็ น Grave disease หมอให้ยำ PTU
มำ 3 เดือน ควรส่งตรวจอะไรเพื่อบอกผลข้ำงเคียงของยำนี้

1. Free T4
2. TSH
3. Blood culture
4. Throat swab culture
5. Complete blood count
13.ญ OP กิน Calcium supplement ต้องระวังกำรรับประทำนร่ วมกับยำใด
1. Ofloxacin(Ans เพรำะธำตุที่มีประจุหำ้ มกินกับ Fluoroquinolones, Tetracyclines,
Thyroxin)

B1.9 Quantitative Methods

1. ถ้ำมค่ำ outlier เข้ำมำในกลุ่มตัวอย่ำง 20 คนแล้วค่ำใดจะเปลี่ยนไปมำกที่สุด


1) Median
2) Mode
3) Mean(Ans)
4) Interquartile range
2. ข้อสอบเก่ำ(คล้ำยๆแกะบอลลี่ขอ้ 3) 10/200 5/200 หำ NNT ของยำ -> 20

3. ให้ขอ้ มูล p-value = 0.006 สรุ ปว่ำอย่ำงไร

1. ….
2. ….
3. สรุ ปไม่ได้ เพรำะ กลุ่มตัวอย่ำงน้อยเกินไป
4. หำ risk factor of VZ from 80 people who has history of VZ and 320
who didn’t has VZ , all have history of chickenpox เป็ น study แบบไหน

a. Case control
b. Retrospective cohort
c. Prospective cohort
d. Clinical trial
5. ถ้ำเพิ่มกลุ่มตัวอย่ำง จะเกิดอะไรขึ้น

1. decrease p-value
2. increase mean of …
3. increase mean of ….
4. decrease power of the test
5. widening 95% confident interval
6. แบ่ง subject ทำครี มกันแดด ครี ม A ทำแขนด้ำนหนึ่ง ครี ม B ทำแขนอีกด้ำน วัดเวลำตั้งแต่เริ่ มทำจนถึงเกิดรอย
แดง ถำมหำวิธีกำรสถิติที่ใช้วเิ ครำะห์
1. ANOVA
2. chi-square test
3. correlation
4. paired T-test(Ans)
5. independent T-test
7. จำกตำรำงแสดงควำมเชื่อมโยงระหว่ำงกำรสูบบุหรี ต่อวัน และ Risk ของ Myocardial Infarction

Cigarettes/day OR of MI

<10 4

10-20 10

>20 20

ตำรำงนี้แสดงถึงอะไร

1. Strength
2. Consistency
3. Biological plausibility
4. Temporal relationship
5. Dose-response relationship(Ans)
8. Surgeon อ่ำนงำนวิจยั ว่ำไม่มีควำมแตกต่ำงระหว่ำงกำรให้ยำ antibacterial ในคนไข้ pre และ post-
operation ถำมว่ำ surgeon คนนี้ใช้หลักฐำนประเภทใด

1. Limited evidence
2. Medium evidence
3. Strong evidence
4. Expert consensus
9. จำกกำรศึกษำ พบว่ำในผูช้ ำยมี incidence ของ ischemic heart disease มำกเป็ น 5 เท่ำของผูห้ ญิง
แต่พบว่ำทั้งผูช้ ำยและผูห้ ญิงมี prevalence พอ ๆ กัน ถำมว่ำเพรำะอะไรถึงเป็ นเช่นนี้

1. More risk factors in male


2. Crude mortality ของ male มำกกว่ำ
3. Case mortality ของ male มำกกว่ำ
4. 5-year survival ของ male มำกกว่ำ
5. กำรดำเนินโรคโดยธรรมชำติของ female ช้ำกว่ำ

B1.10 Microbio --?????

1. เด็กมีอำกำร URI … -> เป็ นหัด เชื้อติดต่อทำงไหน


a. Droplet transmission(Ans)
b. contact transmission
c. Faeco-oral transmission
2. Shellfish ตำ เกิด hemorrhagic bleb, BP drop เกิดจำกเชื้ออะไร
a. E. coli
b. V. vulnificus(Ans)
c. Aeromonas hydrophila
d. K. pneumoniae
3. เล่นน้ ำทะเล เจอ fang mark ->serum CK สูง ควรระวัง complicationใดมำกที่สุด
a. hyperkalemia(Ans from rhabdomyolysis->Renal failure)
b. respiratory failure
c. hypofibrinogenemia
4. หญิงอำยุ 30 มั้ง เป็ นไข้สูง dyspnea pneumonia. what is the causative agent?
1. Streptococcus agalactiae

2. Haemophilus influenzae

3. Streptococcus pneumoniae

5. หญิง 25ปี กิน raw marine fish มีอำกำร intestinal obstruction causative
agents?
1. Ascaris
2. Strongyloid
3. Hookworm
4. Anisakis(Ans)
5. Capillaria
B2 Hemato
1. ผูป้ ่ วยประสบอุบตั ิเหตุ car accident significant blood loss เสี ยเลือดหนักมำก จะมีค่ำใด
ลดลง
1. Ferritin

2. hepcidin

3. Hemopexin

4. Transferrin

5. Erythropoietin

2. A 25-year-old male with subclavian vein thrombosis. Which is decrease


in this man?
1. protein C
2. plasminogen activator
3. antiplasmin
3. Causative agents ที่เป็ น risk ทำให้เกิด lesion ดังภำพ คือ เชื้อ ? (ดูรูป)
1. Cytomegalovirus
2. Epstein-Barr virus
3. Human Papilloma virus
4. Dengue virus
5. Human T-Cell Lymphotropic Virus
4. 5-year-old girl presented with petechiae throughout body. 2 weeks ago
she have fever, cough and rhinorrhea. Blood examination showed Hb 11,
Hct 33, WBC 9000(N 43%, L 47%, M 8%, E 1%, B 1%), Plt 10000. What is
the cause?
1. Deficiency of ADAMTS13
2. Deficiency of platelet glycoprotein IB-IX-V
3. Deficiency of platelet glycoprotein IIB-IIIA
4. Autoantibody to platelet
5. Protein C Deficiency
5. A 35 year-old woman has bleeding after tooth extraction. RBC WBC
8,500( L47%N 42% …) platelet 120,000 PT 10 (10-13), aPTT 50 (32-40).
ristocetin reduce activity. Which blood product should be given to the
patient?
1. cryoprecipitate
2. fresh frozen plasma
3. cryoprecipitate-removed plasma
4. single donor platelet concentrate
5. whole blood product
6.อะไร Initiate Fibrinolysis

1. Plasminogen activator
2. Fibrin kinase
7. ผูป้ ่ วย DM มีค่ำ CBC : Hb ต่ำ และค่ำ อื่นๆ อยูใ่ นเกณฑ์ปกติ, ผลกำรตรวจอื่นพบ BUN, Cr สูง ถำมว่ำ
ผูป้ ่ วยซีดจำกอะไร

1. Epo ลดลง

B3 CNS
1. Ataxia, intention tremor, … เสี ยที่ไหน
a. Cerebellum
b. basal ganglia
c. cerebrum
d. midbrain
e. pons
2. เด็กตกเก้ำอี้ มีแผลที่จมูก เจอ CSF rhinorrhea กระดูกชิ้นใดหัก : cribriform plate,
ethmoid, lesser wing of sphenoid bone clivus
3. Depression เกิดจำก neurotransmitter ใดลด : serotonin, dopa,...(ตรงข้อสอบเก่ำ)
4. Resting tremor, เดินช้ำ? มีพยำธิสภำพที่ใด : substantia nigra
5. มี brain tumor แล้ว เสี ยงแหบ โดนรู ไหน : jugular foramen, foramen ovale,
foramen spinosum
6. has seizure. Her eye deviate to the left side of the body. Where is the
origin of this seizure?
1. left frontal lobe
2. right parietal lobe
3. left temporal lobe
4. right frontal lobe
5. left parietal lobe
7. contralateral hemiparesis + ptosis

1. anterior cerebral artery


2. …
3. superior cerebellar artery
4. anterior inferior cerebellar artery
5. posterior inferior cerebellar artery
8.Median nerve ถูกตัด cell หลังจำกแพทย์ได้ทำกำรผ่ำตัดต่อใหม่ อะไรซ่อมแซมNerve

a. Neuron
b. Schwann cell
c. Microglia
9. ชำย เป็ น migraine จะใช้ ยำที่เป็ น agonist ของ neurotransmitter ใด

1. Dopamine
2. Epinephrine
3. Norepinephrine
4. Serotonin
10 หญิง เป็ น โรค ซึมเศร้ำ มี ควำมผิดปกติ ของ neurotransmitter ใด

1. Epinephrine
2. Serotonin
3. Dopamine
4. GaBA
11. ชำย มี อำกำร resting tremor cogwheel rigidity postural instability มี ควำมผิดปกติ
ที่ ?

1. Medulla oblongata
2. Prefrontal cortex
3. Midbrain
4. Pons
12. ชำยมีอำกำรของ facial nerve palsy + hearing loss lesion น่ำจะอยูบ่ ริ เวณใด

1. Stylomastoid foramen
2. Parotid gland
3. Genu of facial nerve
4. Jugular canal
5. Internal acoustic meatus
13. ผูป้ ่ วย ทำ subtotal thyroidectomy surgeon ผูก inferior thyroid artery ต้องระวัง
nerve อะไร

1. Recurrent laryngeal nerve


2. Internal laryngeal nerve
3. Superior laryngeal nerve
4. External laryngeal nerve
14 surgeon จะ ทำ closure of patent ductus arteriosus จะต้องระวัง structure ใด

1. Recurrent laryngeal nerve


2. External laryngeal nerve
3. Phrenic nerve
15. ผูป้ ่ วยมี……, auditory hallucination พยำธิสภำพอยูท่ ี่ใด

1. frontal lobe
2. occipital lobe
3. temporal lobe
4. superior parietal lobule
16. anencephaly

1. agenesis of neural tube


2. failure of anterior neuropore closure
17. คนไข้ชกั ต่อเนื่องมำห้องฉุกเฉิ น ให้ยำอะไร

1. Diazepam
2. Valproate

B4 Skin
1. คันขำหนีบ ตรวจ KOH preparation ได้ดงั รู ป ถำมว่ำเป็ นโรคอะไร

a. Tinea cruris
b. Malassezia furfur
c. Candida

2. เด็กเป็ น Dermatitis ที่หน้ำ ควรใช้ยำใด

1. clobetasol
2. prednisolone
3. triamcinolone
4. betamethasone
5. Hydrocortisone(Ans low potency topical steroid)
3. clinical ให้เป็ น psoriasis. what is the pathogenesis of the disease?

1. decrease keratinocyte proliferation


2. increase keratinocyte differentiation
3. Th1 and Th17- induced inflammation(Ans)
4. A female ผมร่ วงเป็ นหย่อม แต่ผิวหนังบริ เวณ lesion ดูปกติ พบบริ เวณนั้นมีผมขึ้นควำมยำว 2 -10 mm
จงให้กำรวินิจฉัย

1. alopecia areata
2. Trichotillomania
5. Subbasal bullae pathogenesis เป็ นอย่ำงไร

1. …
2. ….
3. มี autoantibody to hemidesmosome
4. …
5. ….

B5 Musculoskeletal
1. บวม 1st MTP joint Uric 10 กินอะไรปลอดภัย

a. Egg
b. Beef
c. Salmon
d. Asparagus
e. bean sprout
2. Dorsiflexion, eversion เท้ำไม่ได้ nerve อะไรเสี ย

f. Peroneal
g. Sural
h. tibial
i. sciatic
3. Gout แล้วกิน alcohol ทำไมอำกำรถึงแย่ลง(ข้อสอบเก่ำ)

j. Decreased renal uric excretion


k. increase purine intake from liquor
l. decrease purine salvage pathway
m. increase de novo purine synthesis
4. sigmoid colon cancer. Large area sigmoid colectomy ต้องผูกหลอดเลือดใด

1. iliac vessel
2. inferior rectal artery
3. superior rectal artery
4. inferior mesenteric artery(Ans)
5. superior mesenteric artery
5. A man has total hip replacement. The patient has difficulty of walking
(trendelenburg gait). Which nerve is most likely injured from the
operation?
1. obturator nerve
2. femoral nerve
3. tibial nerve
4. sciatic nerve
5. superior gluteal nerve
6. A woman diagnosed with osteoporosis underwent hip replacement.
The subchondral region of head of femur showed wedge-shape
necrosis. What is the pathology?

1. avascular necrosis
2. Granulomatous inflammation
7. disc herniation S2-S4 อะไรป้ องกันไม่ให้ disc herniate

1. supraspinous ligament
2. ligamentum flavum
3. ligamentum nuchae
4. anterior longitudinal ligament
5. posterior longitudinal ligament(ตอบ)
8. OA สำรใดในข้อลด

1. Chondroitin sulfate
9. Pudendal nerve block ทำที่ตรงไหน

1. Obturator foramen
2. Ischial spine
10. Pudendal nerve injury ข้อใดจะใช้งำนไม่ได้

B6 RS
1. A man has progressive dyspnea for 5 years. PE shows engorged neck
vein, hepatomegaly, pitting edema, fine crepitation at both lower lung.
What is the histopathology in his lung?
1. pink amorphous material with few debris
2. plexiform lesion in muscular artery
3. …
4. …
5. extravasated RBC…..

3. A 1 year-old boy presented with coughing in cluster with aggravate in


the night. Physical examination shows no fever and subconjunctival
hemorrhage. He has no vaccination history.
What is the mechanism of the clinical manifestations ?
1. increased mucous secretion
2. Mucosal inflammation
3. Decreased mucociliary clearance(Ans)
4. decreased normal flora

5. pseudomembrane formation

4 กล้ำมเนื้อใด ช่วยเกี่ยวข้องกับ breathing เปิ ด vocal cord

1 Lateral cricoarytenoid muscles


2 Transverse arytenoid muscle
3 Posterior cricoarytenoid muscles(Ans)

4 Sternothyroid muscles
5. ชำย 60 ปี copd, history of smoking ย้อมgram stain จำกsputum พบ

Causative agents?
1 Klebsiella pneumoniae
2. Streptococcus pneumoniae

3 Eikenella corrodens
4. Haemophilus influenzae

5. Pseudomonas aeruginosa(Ans)

6. A 70 year-old man postoperative day 10 has sudden dyspnea. เกิดอะไรขึ้น

1. atelectasis
2. acute pulmonary edema
3. pulmonary embolism(Ans)
7. A female has wheezing. She is given salbutamol but wheezing still
persist. Which drug should be given?

1. montelukast
2. theophylline
3. prednisolone
4. sodium cromoglycate
8. เป่ ำเทียน 20 เล่มใช้กล้ำมเนื้ออะไร (ข้อสอบเก่ำ)

1. external and internal intercostal muscle


2. abdominal and internal intercostal muscle
9. ให้โจทย์มำ สุดท้ำยบอกมี forced air inspiration เป็ นผลจำกกล้ำมเนื้อใด

1. Pectoralis major
2. Pectoralis minor
3. Scalene
4. Sternocleidomastoid(Ans)

B7 CVS
1 แฝด 2 คน มำ ออกกำลังกำยด้วยhrเท่ำกัน แต่o2 consumptionต่ำงกัน. อะไรต่ำงกัน

ทำไมต่ำงกัน. 1. sympathetic activity 2. Stroke volume 3 tissue O2 extraction. 4


mitochondria.

2. A 70 year-old male with chronic hypertension. He suddenly fell


numbness in his leg and his toe is cold. Normal ECG. Where is the
arterial thromboembolism source in this patient?
1. paradoxical embolism
2. thrombophlebitis
3. Atherosclerotic plaque
4. aortic aneurysm?
5. mural thrombus in left ventricle
3. Mycotic aneurysm. A patient has middle cerebral artery occlusion.
What is the cause?
1. acute rheumatic fever
2. infective endocarditis
4. A boy has pulmonic valve stenosis, dextroposition of the aorta and
ventricular septal defect. What is the most likely pathology to be found in
this boy?
1. ectopia cordis
2. …
3. right ventricular hypertrophy
5. สูบบุหรี่ นิ้วขำดเลือด gangrene สำเหตุคือ

1. direct endothelial injury


2. ….
6. (ข้อสอบเก่ำ 2015) aortic stenosis BP 170/50 mmHg อะไรทำให้ pulse pressure
สูง

1. ….
2. decreased arterial compliance
3. …
4. ..
5. increased peripheral vascular resistance

B8 GI
1. Blood accumulation when supine at?

1. Rectouterine pouch

2. Hepatorenal pouch(Ans)

3. Rectovesical pouch
4. Hepatoduodenal pouch

2. ปวดหัว กิน land snail → ปวดหัวหอย ซ้ ำข้อสอบเก่ำครับ

3. ชำวญี่ปุ่น ปวดท้องมีประวัติกินปลำทะเลดิบ ให้รูปมำ

ถำมว่ำติดพยำธิอะไร -> anisakis, ascariasis

4. มี hernia ทะลุผำ่ น superficial inguinal ring วินิจฉัยเป็ น Direct inguinal hernia ถำมว่ำ
structure ใดที่ hernia นี้อยู่ medial ต่อ (คือถำมว่ำอะไรอยู่ lateral ต่อ structureที่ herniate)

a. Lateral umbilical ligament


b. Spermatic cord
c. inguinal
5. RUQ pain, Ultrasound show thickened gallbladder wall.
Cholecystectomy was performed. There are 10 greenish-black stones,
0.5 - 1 cm in diameter. What is the underlying disease of this patient?

1. Hypercholesterolemia
2. G-6-PD deficiency
3. Hereditary spherocytosis
6. A patient was diagnosed with ulcerative colitis.A drug was given to
him. He develops nausea, vomiting,..... and mucositis. Which substance
prevents this side effect.

1. leucovorin
2. Folic acid
3. pyridoxine
7. A man has substernal pain especially after eating a meal in the
evening. Which drug can relieve the symptoms?

1. ondansetron
2. omeprazole
8. a 50-year-old man who had COPD presents with the
pitting leg edema for 3 months
PE : engorged neck vein and hepatomegaly
what is the following is the main pathological change to the liver?
1. necrosis
2. infarction
3. congestion
4. fatty change
5. bridging septa

9.อะไรบ่งบอกpoor prognosis ของ acute pancreatitis

1.hypocalcemia

B9 KUB
1. A 20 year-old man has recurrent hematuria. RBC 5-10/hpf What should
be the history leading to his symptoms?
1. ….

2.

3. Bacterial pharyngitis 2 weeks before first hematuria episode

4. upper respiratory tract infection 2-3 days before

5. Bacterial skin infection and hematuria episode

2. 25-year-old has edema. UA: proteinuria 3+, RBC 50-100/hpf. What is the
most likely diagnosis?
1. minimal change disease
2. focal segmental glomerulosclerosis
3. diffuse proliferative glomerulonephritis
4. acute poststreptococcal glomerulonephritis
5.
3. สะดือแฉะ has content from umbilicus

1. omphalocele
2. gastroschisis
3. umbilical hernia
4. persistent vitelline duct
5. persistent allantois lumen(Ans)
4. flank pain radiate to inguinal area and scrotum.(รอบบ่ำย)

1. T10-T12
2. T12-L2
3. L2-L4
4. L4-S1
5. A 5 year-old boy has hematuria and sensorineural deafness. Which
type of collagen is mutated in this patient?(รอบบ่ำย)

1. I
2. II
3. III
4. IV(Ans Alport)
5. V
6. has hematuria. renal biopsy shows necrotizing renal …. พบ antibody to
myeloperoxidase. Skin biopsy ควรพบอะไร (รอบบ่ำย)

1. leukocytoclastic vasculitis
2. lymphocytic infiltration at dermis
3. necrotizing granulomatous vasculitis
7.คนไข้เป็ นอะไรซักอย่ำง Naขึ้น Hypokal ถำมว่ำHypokalemia เกิดจำกCellใด

1. Principal cell
2. Intercalated cell
8. นอนหลับกลำงคืน แล้วตื่นเช้ำมำปั สสำวะน้อย ถำมว่ำเกิดจำกอะไร

1. ADH เพิ่ม 2. Renin เพิ่ม 3. Epinephrine เพิ่ม 4. Aldosterone ลด 5.


Angiotensin II ลด
9. กิน protein เยอะๆ แล้วปัสสำวะมำก เกิดจำกอะไร

1. Urea มำก
2. จำไม่ได้

3. Plasma Na conc เพิ่ม

4. Renin ลด

5. Renal Na absorption ลด

B10 Repro
1. ให้รูป hypospadias มำ ถำมว่ำ structure ใดผิดปกติ
1. urogenital fold(Ans)

2. cloacal membrane

2. placenta previa. blastocyst implant ที่ไหน

1. uterine tube
2. uterine horn
3. uterine fundus
4. uterine body
5. internal os (Ans)
3. A female (อำยุอยูใ่ นวัยเจริ ญพันธ์ จำไม่ได้ 555) ก้อนขอบเขตชัด ตัดออกง่ำย ก้อนไม่แน่นมำก พบมีท้ งั
stromal, fibrous proliferation และ duct epithelium proliferation.(ข้อสอบพูดยำวกว่ำนี้
แต่สรุ ปได้วำ่ มีท้ งั stromal และ epithelial proliferation) What is the diagnosis?

1. galactocele
2. fibroadenoma
3. Phyllodes tumor
4. invasive ductal carcinoma
5. lactating adenoma
4. กิน oral contraceptive pills เสี่ ยงต่อกำรเกิดโรคใด

1. …..
2. …..
3. Cervical cancer
4. Ovarian cancer
5. Labia majora มี cancer จะพบ cancer กระจำยไปที่ต่อมน้ ำเหลืองใดเป็ นตำแหน่งแรก
1. Internal iliac node
2. External iliac node
3. Sacral node
4. Lumbar node
5. Superficial iliac node
B11 Endocrine
1. กินน้ ำก่อนนอนไป 1L ทำไมตื่นมำถึงฉี่ นอ้ ย?
1. increase ADH, 2. decrease sympathetics 3.increase aldosterone

2. patient has history of polydipsia, polyuria. She has high serum


osmolarity and urine osmolarity 80 Osm/L. Serum arginine vasopressin is
high. What is the diagnosis?

1. primary polydipsia
2. central diabetes insipidus
3. nephrogenic diabetes insipidus
4. syndrome of inappropriate ADH secretion
3. โจทย์คล้ำยๆ ข้ำงบน ตัด serum AVP is high ออก ถำม Which investigation is used for
the diagnosis?
1. DDAVP test
4. เด็กตัวใหญ่ 97th percentile pubic hair but small testes. What should
increase in this boy?
1. cortisone
2. 17-hydroxyprogesterone
5. Danazol เป็ น synthetic ของ hormone ใด

1. Estrogen
2. Progesterone
3. Androgen(Ans)
6. ผูห้ ญิง ได้รับยำคุม ชนิดหนึ่งทำง IM every 12 weeks เพื่อป้ องกันกำรตั้งครรภ์ ยำคุมนั้นเป็ น
synthetic ของ hormone ใด

1. Estrogen
2. Progesterone(Ans)
3. androgen
7. กิน prednisolone Sputum มี larva จะติดเชื้ออะไร
1. Strongyloides stercoralis
8. DM type 2 plasma osmolarity 320 Osm/L พบ ketone bodies อะไรทำให้
plasma osmolarity สูง

1. ketone body
2. gluconeogenesis

รอบบ่ำย

B1.1.Biochem & Molecular Genetics

1. A 20-year-old woman participated in 100-meter running race. After


the race, her blood pH reduced from 7.42 to 7.25. Which of the
following component responsible for reduced blood pH?
A. Alpha oxidation

B. muscle glycogenolysis

C. anaerobic glycolysis

D. creatine phosphokinase

2. 2. Strict vegetarian มีอำกำรเหนื่อยง่ำยจำกกำรขำดอะไร


a. Cobalamine
3. น้ ำตำลสูง plasma osmol 320 osmol สูงขึ้นจำกอะไร
a. Gluconeogenesis

B1.2.Biology of cells

1. ตรวจพบ BCL-2 over expressionถำมว่ำกระบวนกำรใดจะผิดปกติ


A. apoptosis

B1.3.Human development and genetics

1. Pedigree mitochondrial inheritance


B.1.4.Normal immune response

1. เด็กถูกตีกน้ แดง แดงเกิดจำกผลของสำรใด


a. Bradykinin
b. Serotonin
c. Nitric Oxide
d. histamine
2. หญิง 20 ปี เกิดก้อนผิดรู ปที่จมูกหลังไปฉีดซิลิโคนมำ 2 สัปดำห์ เซลล์ใดที่ส่งผลให้เกิดควำมผิดปกติดงั กล่ำว

a. Basophil
b. Eosinophil
c. Neutrophil
d. Macrophage
3. Rabies ทำไมต้องฉี ด vaccine หลำยรอบ

→ Stimulating sufficient neutralizing antibody

→ ช้อยที่เหลือคล้ำยๆปี ที่แล้ว

B.1.5 Pathogenesis , Pathophysiology , Basic Pathologic process , LAB


interpretation
1. เด็ก สองสัปดำห์ก่อน เป็ นsore throat แล้วเป็ นpost-strep(Nephritis) ส่งตรวจอะไร?

a. anti-DNAse B
b. Throat swab culture
c. hemoculture
2. ญ 30 y Facial rash, joint pain, photosensitivity, อะไรอีกซักอย่ำง
อะไร specific กับโรคนี้ที่สุด

a. anti-dsDNA
b. anti-ssDNA
c. Antinuclear antibody
d. anti-small RNP
e. Anti-histone
3. CBC เจอE10 ตรวจอะไรเพิ่ม

a. Stool examinations
4. ตรวจ antibody ต่อไวรัสแล้วส่งไม่ได้ให้ทำไง(ข้อสอบเก่ำ)

a. แช่ 4 c
B.1.6. Gender,ethics , behavioral consideration affecting disease
treatment , and prevention including psychosocial cultural and
environment
1. Alloy factory, found bubble urine and beta2-microglobulin in urine
เพรำะสำรอะไร
A. Cd
B. As
C. Mn
D. Pb
E. Hg

2. ชำยอำยุ 50 ปี เป็ นโรคควำมดันสูงมำ 10 ปี ปั จจุบนั ใช้ยำและสำมำรถควบคุมควำมดันได้ดี วันนี้ชำยคนนี้มำพบ


แพทย์เพื่อต้องกำรให้แพทย์ออกใบรับรองแพทย์เพื่อใช้ในกำร Early retirement แพทย์ควรออกให้หรื อไม่
เพรำะเหตุใด

1. ควร เพรำะ patient beneficence


2. ควร เพรำะ patient autonomy
3. ควร เพรำะ physician duty
4. ไม่ควร เพรำะ unethical conduct
5. ไม่ควร เพรำะ injustice
3. เด็กติดเกมสถำบันไหน?

B1.7.Multisystem process

1. เด็ก type 1 DM, Bp drop, bl. Sugar 400 จะให้สำรน้ ำอะไร

1. dopamine IV

2. dobutamine IV

3. dextran IV

4. Mannitol IV
5. NSS IV

2. Male has blurred vision after drink home-brew liquor which is most
appropriate treatment
1. Ethanol
2. Disulfiram
3. Ethylene glycol
3. ผูป้ ่ วยเด็ก ซื้อยำลดไข้มำกินเอง ช่วงแรกเกิด Hyperventilation แต่พอมำตรวจแล็บพบ pH 7.3 HCO3
12 PCO2 20 PaO2 95
1. metabolic acidosis
2. respiratory alkalosis
3. compensated metabolic acidosis
4. mixed metabolic acidosis and respiratory alkalosis
(Aspirin/salicylate toxicity)
4. 30-year-old diabetic woman presented with fever, hyperpnea and
drowsiness. Her blood examination revealed FBS 400, Na 135, Cl 100, K
5.7, HCO3 15. What is the main mechanism of hyperkalemia?

1. Hyperglycemia
2. Metabolic acidosis
3. Respiratory alkalosis
4. Decreased K excretion
5. Shift of K to extracellular

B1.8. General pharmacology

1. 70-year-old man presents with urinary incontinence and has low


urine stream. Doxazosin is prescribed by his physician. What
changes may occur in this patient?
A. Diarrhea

B. Bradycardia

C. Impotence

D. Postural hypotension
E. Premature ejaculation

คำตอบ D. Postural hypotension

คำอธิบำย Doxazosin มีฤทธิ์ block alpha1 receptor ทำให้เกิด vasodilatation เกิด


Postural hypotension ตำมมำได้

2. Organophosphate poisoning ให้ atropine อำกำรใดยังอยุ่ (ช้อสอบเก่ำำ)

a. Muscle fasciculation
B1.9 Quantitative methods

1. ยำ A ลด 50% ยำ B ลด 50 % p 0.006 ข้อใดถูก


1. rrr 40%
2. Arr 25
3. ไ่่ม่มี stat sig
4. RR ลด 1.4 times
2. ผูป้ ่ วย DM เป็ น neuropathy ไม่ให้ยำโอกำสเป็ นโรค 10/100 ให้ยำ X โอกำสเป็ นโรค5/100
ถำม NNT
B1.10 Microbio

1. 10-year-old boy presented with sore throat and rashes on trunk. His
tongue is swollen and rough, look as red as strawberry. What is the
causative organism?
a. Measle virus
b. Rubella virus
c. Bordetella pertussis
d. Staphylococcus aureus
e. Streptococcus pyogenes

B2 Hemato
1. ผูป้ ่ วย active TB มี anemia (Anemia of Chronic inflammation) เป็ นผลจำกสำรใดเป็ น
หลัก

1. Hepcidin
2. transferrin

3. hemopexin

2. ปกติทุกอย่ำง แค่ WBC มี eosinophil สูง ควรส่งตรวจอะไร

1. osmotic fragility test


2. stool examination
3.Occasionally dark urine on morning

1. Spectrin
2. จำช้อยส์ไม่ได้
4. ญ 20 ปี Hb 9 Hct 27 reticulocyte count 1.5 % WBC ปกติ plt 20,000 ควรส่งแลป
อะไร

a. Coomb
b. bm biopsy
c. bone scan
d. ct or mri
5. ให้ Heparin ถำม monitor อะไร

B3 CNS
1. ให้รูปนี้มำ brain infarct จะมี neurological deficit อย่ำงไร

ถำมอำกำร 1. contralateral hemiparesis at face

2. contralateral hemiparesis at arm

3. contralateral hemiparesis at leg


4. ipsilateral

1. ขำอ่อนแรงมำกกว่ำแขน
2. แขนอ่อนแรงมำกกว่ำขำ
3. อ่อนแรงเท่ำกันทั้งตัว
4. อ่อนแรงทัว่ ตัวมำกกว่ำขำ
5. อ่อนแรงทัว่ ตัวมำกกว่ำแขน
2. จะblock pudendal nerve landmark อยูต่ รงไหน

A.ischial spine

B.ischial tuberosity

3. car accident ptosis and impaired lateral gaze. กระดูกอะไรแตก

1. lesser wing of sphenoid bone

2. ethmoid bone

3. Optic canal

4. Clivus

4. resting tremor, bradykinesia ขำด enzyme ใด

1. dopamine decarboxylase
2. tyrosine decarboxylase
5. Anencephaly which is defect

1. Anterior neuropore
6. Continuous convulsive what is the first treatment

a. Valproate
b. Diazepam
c. Phenytoin
7. Migraine เวลำโดนแสงต้องใช้ยำกระตุน้ receptor ใด

a. Serotonin
8. A 50-year-old man is presence with blurred vision after drinking
distilled home-brew liquor.
best treatment?
a. Ethanol
b. Thiamine
c. Disulfiram
d. …
e. ethylene glycol
9.

B4. Skin

1. โรค Harlequin ichthyosis เกิดจำกควำมผิดปกติที่ใด


A. Keratinocyte
B. basement membrane
2. The 40 year-old woman complain about stiffness of finger PE: round
furrow around mouth and skin biopsy show absent of skin appendage
presence thick collagen fiber w/ atrophic epidermis.What is most
common clinical present in this patient ?
A. Urolithiasis

B. Dysphagia

C. Hypermenorrhea

D. Acne

3. The 5 year-old boy have severe itchy friend and brother also have this
symptoms too.PE: mutiple vesicle on erythematous base with scratch
marks on hand, feet and buttock.ถำมว่ำเป็ นโรคอะไร

A. Pediculosis
B. Scabiasis
C. Impetigo
4. ผูห้ ญิงตำกแดดนำนแล้วหน้ำบวมแดงถำมว่ำเกิดพยำธิสภำพที่ใด

A. epidermis
B. papillary dermis

C. reticular dermis

D. subcutaneous tissue

5. ตำกแดดนำน ผิวเข้มขึ้น เกิดจำก enzyme ใด

1. Tyrosinase
2. Valine hydroxylase

B5 : Musculoskeletal system

1. ผูใ้ หญ่เป็ นgout ให้ Colchicine มีADRsอะไรที่ตอ้ งระวัง?


a. Nausea
b. Diarrhea
c. Rash
2. ผูป้ ่ วยหญิงมีอำกำร proximal muscle weakness, มี ptosis ตอนให้กระพริ บตำหลำยๆ
ครั้ง เกิดจำก

1. Autoantibody ต่อ ACh receptor,...


3. ผูห้ ญิงปวดข้อเข่ำ เป็ น RA ถำมว่ำเกิดจำกอะไร

Antibody to Fc portion of IgG, Antibody to Fab portion of IgM


4. ผูป้ ่ วยผ่ำตัดเปลี่ยนข้อสะโพก difficulty in walking (trendelenburg gait) nerve ใด
มีโอกำส injury มำกที่สุด

1. Superior gluteal nerve


2. Femoral nerve
3. Sciatic nerve
4. Obturator nerve
5.
4. Ankle sprain ฝั่งMedial เกิดกำรอักเสบที่ใด

1. Deltoid ligament
2. Tibialis anterior tendon
3. Talofibular ligament
5. 15 years old boy bone tumor @distal fibula มีกำรสร้ำงosteoid เป็ นอะไร
→ Osteosarcoma

→ Chrondrosarcroma

- …

B6 RS
1. A 2-year-old did not get what he wanted. Then, he screamed, cried
and held his breathe for about one minute and turned blue.
Afterwhile, he rebreathed again. What is the most important
stimulator causing him to rebreathe again?
a. PaO2
b. PaCO2
c. Acidosis
d. Alkalosis
e. 2,3-bisphosphoglycerate
2. AIDs หอบเหนื่อย… ให้รูปนี้มำ (รู ปนี้จริ งๆนะ)

ถำมเชื้อก่อโรค -> pneumocystis jiroveci

3. เด็กจมน้ ำ ปอดเป็ นอย่ำงไร (ข้อสอบเก่ำ)

1. decreased elastic recoil


2. decreased surface tension
3. increase pulmonary blood flow
4. increased dead space
4. A xx year-old man has history of smoking presented with dyspnea.
CXR เป็ น Patchy infiltration at right lower lobe ให้ภำพ gram stain เป็ น gram
negative rods (ภำพดูไม่ชดั ว่ำมี capsule หรื อไม่)

What is the causative agent?


1. Klebsiella pneumoniae
2. Pseudomonas aeruginosa
(อุปส์...เจอรู ป Searchว่ำ Pseudomonas aeruginosa sputum

http://www.fujita-hu.ac.jp/~tsutsumi/case/case071.htm
*ข้อนี้เป็ น Bronchopneumonia (จำกCXR) ไม่ได้เป็ น Lobar จึงไม่ควรเป็ น Klebsiella ครับ :3
+Smoking เป็ น risk for community acquired Pseudomonas Pneumonia
ปล.อำจดูได้จำกกำรที่มนั เป็ น Negative rod ตัวยำวๆ แต่แยกยำกเกิ๊น =..= )

5.ผู้่ป่ วย COPD, pursed-lip breathing, Barrel chest มีกลไกอย่ำงไร

1. Imbalance of elastase and anti-trypsin


6. เสมหะมี neutrophil, foul smell ทำ CT (หรื อ CXR จำไม่ได้) พบ consolidation at right
lower lung. bronchoalveolar lavage พบ debris, inflammatory cell, muscle
cell, adipose cell with some microorganism คนไข้เป็ นอะไร

1. atelectasis
2. lung abscess
3. lobar pneumonia
4. bronchiectasis
5. Foreign body aspiration
7. During the inspiration, which muscle is working to seperate out the two
sides of vocal cords to let the air pass trought?
1. Cricothyroid
2. Thyroerytenoid
3. oblique erytenoid
4. transverse erytenoid
5. posterior cricoerytenoid
8. A 10-year-old boy is unconscious after an episode of near drowning

most likely change of his lung?


a. ⬇️ elastic recoil
b. ⬇️ surface tension
c. ⬇️ surfactant synthesis
d. ⬇️ alveolar dead space
e. ⬇️ pulmonary blood flow

B7 CVS
1. Patient with pulmonary valve stenosis. the ECG show right ventricular
strain pattern. ถำมว่ำ mean electrical axis of ECG จะเป็ นไง
1.-100 to -50

2. -50 to 0

3. 0 to +50

4.+50 to +100

5. +100 to +150

2. A 60 year-old man (ควำมดันปกติ) presented with complete heart block and


systolic ejection murmur radiating to the neck while playing golf. What is
the pathology?
1. dissecting aorta
2. calcified aortic stenosis
3. bicuspid aortic valve
3. A man faint heart sound. engorged neck vein. ECG ให้รูปมำแค่V1

What is the diagnosis?


1. cardiac tamponade
2. hemothorax
3. atelectasis
4. pneumothorax
4.ชำย 40 ปี สูบบุหรี่ ปวดน่อง2ข้ำง ปลำยเท้ำไม่รู้สึก วินิจฉัย?

1. Thromboangiitis obliterans (คล้ำยข้อสอบเก่ำ)


2. polyarteritis nodosa
5. man 70 , Chronic hypertension ,ไม่มี pulse ที่ distal ,arterial embolism แบบ
ใด
1.Left ventricular mural emboli
2.deep vein thrombosis
3.atheromatusplaque
4.peridoxal thromboli
5.thrombophebitis

6. Na=140,K=6,Cl=100 ตรวจEKG พบอะไร


1...P wave
2.ST depression
3.Tall peak T wave
4.Prolongation of QT interval
5.Prolongation of ... Interval

7.EDV 200 ESV 160


EF=?
1.5
2.10
3.20
4.25
5.30

B8 GI
1. AIDs + chronic watery diarrhea microscopic examination shows

ขนำด 8-10 um
1.cyclospora, 2. Cryptosporidium 3. Isospora 4 microsporidia

2. 60 years old women, lymphadenopathy,increase thickness wall of


stomach, มี Cachexia ขยำยไม่ค่อยได้(ไม่แน่ใจ) ลักษณะจุลจะเจออะไร :signet ring cell,
keratin pearl + intercellular bridge, well-diff gland(ช้อยส์น้ ีไม่แน่ใจ),diffuse
fibrosis
3. Chemo -> mucous bloody diarrhea เกิดจำกเชื้อใด : C. difficile
4. Eat raw beef เสี่ ยงเป็ นอะไร
a. Taeniasis
b. Sparganosis
c. Spirometra
d.
5. Risk factor of SCC esophagus
a. Smoking
b. Barrett esophagus
c. Coffee drinking
6. Accident -> NG tube มี coffee-ground ของเหลวดำๆออกมำ -> ในกระเพำะเจอ
erosion เป็ นอะไร
a. Peptic ulcer
b. Acute gastritis
7. cirrhosis เซลล์ใดทำให้เกิด liver fibrosis (จำคลินิคอลที่โจทย์ให้มำไม่ได้)

1. kupffer cell
2. hepatic stellate cell
8.คนไข้อำกำร confused serum ammonia สูง ตรวจเป็ น cirrhosis วินิจฉัย?

1. Hepatic encephalopathy
2. Wernicke encephalopathy
9.Hepatic encephalopathy ให้ยำอะไร

1. Lactulose
10 เด็กทำรกหญิง ไม่มีรูanus มีอจุ จำระไหลทำง vagina โครงสร้ำงใดผิดปกติ

1. Cloacal membrane
2. Urorectal Septum
3. Proctodeum
4. Distal hindgut (ตอบอันนี้เพรำะทั้ง ไม่มีรูเปิ ด และเกิดfistula)
11.ลดควำมอ้วน ทำนhigh protein, low carb ฮอร์โมนอะไรเพิ่ม

1. Secretin
2. Glucagon
3. Cholecystokinin
12.ปวดท้อง Middle Upper Quadrant, amylaseขึ้น กำรขึ้นของระดับอะไรบ่งPoor
prognosis
1. Hypocalcemia (ตอบข้อนี้)
2. Hypertriglyceridemia
13. has pain radiated to the back. high serum amylase. found white chalk
on the omentum เรี ยกว่ำเป็ นปฏิกิริยำอะไร (คล้ำยๆ ข้อสอบเก่ำซักปี )

1. Saponification
2. White infarct
14. เป็ น GERD -> the most effective

1. Omeprazole
2. Alum milk Male
15. 25 year old has acute RUQ -> acute cholecystitis เจอนิ่วก้อนเล็กๆ 10 ก้อนถำม
underlying disease
1. G6PD def
2. HS
3. Hypercholesterolemia

B9 KUB
1. a man with history of toluene sniffing for 10 years developed
proximal muscle weakness.
PE : proximal muscle weakness 3/5

Blood chemistry : pH 7.25 Pco2 30 PO2 100 BUN 20 Cr 1.2 Na 136


K 2 Cl 112 HCO3- 20

what happen
a. interstitial nephritis
b. renal tubular acidosis
c. acute renal necrosis
d. primary hyperaldosteronism
e. hypokalemic periodic paralysis

2.มีไข้ เจ็บคอ เป็ นมำ 2 weeks urine:RBC, Protein 2+ เป็ นอะไร

A. post-streptococcal glomerulonephritis
B. drug-induced glomerulonephritis
3. เด็ก อำยุ 3 ขวบ hx of recurrent urinary tract infection ปวดบริ เวณ CVA angle ทำ
imaging พบ

Renal pelvis and ureter dilatation อำกำร ของเด็กน่ำจะเกิดจำก โรคใด

1. Vesicoureteral reflux
2. Multicystic renal
dysplasiahttp://emedicine.medscape.com/article/982560-
overview
3. Medullary cystic kidney disease
4. Autosomal dominant polycystic kidney disease
4.ผูป้ ่ วยหญิงคลำได้กอ้ นในท้องขนำดใหญ่ 2 ข้ำง UA: protein 1+, RBC 1-5 /HPF มีควำมเสี่ ยงเป็ น
อะไร(ADPKD)

1. Subarachnoid hemorrhage from Berry aneurysm


5. กิน cloxacillin แล้วแพ้

1. interstitial eosinophilic nephritis


2. interstitial neutrophilic nephritis
6. A female use carbonic anhydrase inhibitor for glaucoma treatment.
She developed metabolic alkalosis. What is the site of renal tubule does
this drug affect?
1. proximal tubule
2. collecting duct
3. distal tubule
4. loop of Henle
7.คนไข้ Hypercalcemia ให้ยำอะไรเพื่อรักษำได้

1. Furosemide
2. Thiazide
3. Acetazolamide

B10 Reproductive system


1. สวนท่อปั สสำวะ แล้วทะลุ พบว่ำ scrotum, anterior abdominal wall ไม่มีกำรบวม
โครงสร้ำงอะไรที่ทำให้ urine ไม่ไหลออกไปข้ำงนอก
A. Buck fascia
B. Colles fascia
C. Dartos fascia
D. Scarpa fascia
E. Camper fascia
2. Straddle injury from motorcycle accident. Radiographic show
fracture of inferior pubic ramus. Which artery is severed?
A. Inferior rectal a.
B. Posterior scrotal a.
C. Internal pudendal a.
D. Obturator a.
E. Deep artery of penis
3. Landmark for pudendal n. Block
a. ischial spine
b. Ischial tuberosity
4. Uterine prolapse which structure is weaken?
a. perineal body
b. transverse perineal muscle
c. Transverse cervical ligament
d. Broad ligament
5. Male has enlarge breast, eat herbal medicine which is in herbal
medicine
a. estrogen
b. Testosterone
c. FSH
d. beta-hCG
6. Labial SCC มี lymph drainageไปที่ไหน
a. Superficial inguinal node
b. Deep inguinal node
c. Internal iliac node
7. ผูป้ ่ วยชำย มี tumor metastasis มำที่ บริ เวณใกล้กบั inferior rectal artery ใต้ต่อ pectinate
line ถำมว่ำ จะเจอ lymph node ใด โต (รอบเช้ำ)

1. Deep inguinal node


2. Superficial inguinal node
3. Lumbar node
4. Internal iliac node
5. External iliac node
8. ผูป้ ่ วย หญิง มำด้วยก้อนที่ขำหนีบ มี external genitalia เหมือน ผญ no ambiguous
chromosome typing พบ 46, XY ทำ imaging พบว่ำ ไม่มีuterus เป็ นโรคใด

1. 5-alpha reductase deficiency


2. 17-alpha hydroxylase deficiency
3. 21-alpha hydroxylase deficiency
4. Androgen insensitivity syndrome

9. ผญ มีประจำเดือน มำวันที่ 10 Jan 2015 cycle 28 วันสม่ำเสมอ จะมีกำร implantation วันไหน

1. 17 Jan 2015
2. 24 Jan 2015
3. 31 Jan 2015
4. 7 Feb 2015
5. 14 Feb 2015

10. Endometrium ที่มีลกั ษณะ prominent coiled gland จำนวนมำก with prominent
mitotic activity and pseudostratification; dense stroma เป็ น endometrium
แบบใด

1. Secretory endometrium
2. Early proliferative endometrium
3. Late proliferative endometrium
4. Menstrual endometrium

11. BPH ให้ยำ ออกฤทธิที่Enz ใด

1. 5-alpha reductase

12. Bone Metas Prostate cancer ตรวจอะไรขึ้นในภำวะนี้

1. PSA

2. ALP
13.หญิง มำพบแพทย์ดว้ ยอำกำรปั สสำวะแสบขัด มีหนองไหลออกมำจำกช่องคลอดมีกลิ่นเหม็น นำไปส่องด้วยกล้องเห็น
ดังภำพ(ภำพ clue cell) เกิดจำกเชื้ออะไร

1.Candida albicans

2.Trichomonas vaginalis

3.Bacterial vaginosis

4.Chlamydia trachomatis

5.Neisseria gonorrhoea

14. กลไกของ Danazol

1. Inhibit aromatase enzyme


2. Decrease LH and FSH
15. Endometriosis ให้ danazol

1. Androgen receptor agonist


16. ยำคุมIM จับกับreceptor ใด

1. Progesterone
17. ให้ EE ในpost menopause ป้ องกัน osteoporosis เพิ่ม risk อะไร

1. CA breast
2. Ovary CA
18. ประจำเดือนขำดมำหลำยเดือน ปกติมีรอบเดือน 28 วัน ตรวจ HCG +ve มีประจำเดือนครั้งสุดท้ำยวันที่ 10
jan blastocyst ฝังตัววันที่เท่ำไหร

1. 24 jan
2. 31 jan
19. Neonatal female external genitalia มี abdominal mass suspects to be
testis. Karyotype is 46 XY เป็ นอะไร

1. 5 alpha reductase def


2. complete androgen insensitivity syndrome
20. เด็กชำย 8 ขวบสูง >P97 เสี ยงแตก เจอ pubic hair , penis enlargements , small
testis เกิดจำกอะไร?
1. Somatotropin
2. Cortisol
3. Corticotropin
21. BPH ได้ยำลดอะไร

1. 5 alpha reductase
22. BPH ได้...osin ->ADR

1. Hypotension
23. Adenoma Dysmenorrhea พบmultiple nodule + right adnexal mass

1. Endometriosis
2. Metastasis adenocarcinoma
24. มี vesicle on erythematous base ตำมมือเท้ำ พี่กบั เพื่อนที่ รรก็เป็ นถำมว่ำเป็ นโรคอะไร (ข้อสอบ
เก่ำ)

a. Scabies

B11: Endocrine system

1. 30-year-old woman, tired, cold intolerance, anti thyroglobulin +, TSH


decrease what is a differential diagnosis
A. Graves disease
B. Hashimoto thyroiditis
2.What is the mechanism of metformin

A. PPAR alpha
B. block K channel
C. inhibits hepatic gluconeogenesis
3.ผูห้ ญิง ควำมดัน 150/100 palpitation urine VMA+ve left suprarenal mass เป็ นโรค
อะไร --->

1. pheochromocytoma

2. metastatic carcinoma

4. Thyroid โต เจำะเจอ psammoma body -> papillary carcinoma, follicular


carcinoma, follicular adenoma
5. Male มี galactorrhea คิดว่ำเกิดจำกสำเหตุใด

1. Increase estrogen

2. dopaminergic antagonist medication

6. A 30 year-old female has progressive dyspnea, palpitation, sweating


and weight loss for 2 months. Physical examination shows T 39C, P
130/min, irregular enlarged thyroid gland. Electrocardiography suggests
atrial fibrillation. The patient is treated with propylthiouracil, Lugol’s
solution and dexamethasone. Which of the following mechanism in
thyroid hormone synthesis is primarily inhibited by Lugol’s solution?

1. coupling
2. deiodination
3. iodine uptake
4. organification
5. colloid resorption
7. ่ืperform total thyroidectomy. After that she feels numbness. ….. (signs
of hypocalcemia). ตรวจเลือดพบอะไร

1. hypercalcemia
2. hyperphosphatemia
3. hypermagnesemia
8. ผูห้ ญิงเป็ นไข้ เคยdiag ว่ำเป็ น graves diseases กิน PTU มำเดือน ตรวจอะไรเพิ่ม

a. CBC ผูป้ ่ วย
9. คนไข้ DM มีรำขึ้นจมูก(จำโจทย์จริ งไม่ได้) รำที่เจอ species อะไร

a. Candida
b. Rhizopus

B1.1 Biochemistry and molecular biology

1. ผูช้ ำยอำยุ 25 ปี มีtendon xanthoma at elbow เกิดจำกกำรสะสมของสำรใดในร่ ำงกำย


A. Chylomicron
B. VLDL
C. ILDL
D. LDL
E. HDL

2. A male patient presents with resting tremor, pill rolling hands, and
slow movement. Which of the followings is most likely deficient in
this patient?
A. Monoamine oxidase
B. Tyrosine hydroxylase
C. Dopamine hydroxylase
D. Tryptophan hydroxylase
E. Glutamic acid decarboxylase

3. 5 years old boy brought to the hospital by his mother due to


smaller size than others and bang something in the dark
History : frequent pulmonary infection and diarrhea

Usually walk and bang things at night


Which of the following is deficiency in this boy"
A. Retinol
B. Ergosterol
C. Tocopherol
D. Menaquinone
E.

4. 5 year old anemia leukopenia thombocytopenia bone marrow


biosy พบ lysosomal storage disease พบอะไร
A. Glycolipid
B. Glycoprotien
C. Sphingolipid
D. Phospholipid
E. Glycogen

5. ผูช้ ำยอำยุ 30 ปี ไปตีกอล์ฟนำนมำก จนผิวดำ ถำมว่ำผิวดำเกี่ยวข้องกับสำรใด


A. Tyrosinase
B.
C.
D. Metalloprotein
E. Homogentisic acid

B1.2 Biology of cell

1. suprabasal seperation of keratinocyte อะไร defect


A. gap junction
B. desmosome
C. zona adherens
D. macula adheren
E. hemidesmosome

2. เซลล์อะไร response ต่อ liver fibrosis


A. Kuffler cell
B. Hepatic stellate cell

B1.3 Human development and genetic

B1.4 Normal immune response

1. ชำยตบแมลง หลังจำกนั้นเกิด erythematous vesiculopapule มี อำกำร burn


sensation ร่ วมด้วย เกิดจำกกลไกภูมิคุม้ กันใด
A. Innate immunity
B. Anaphylactic
C.
D. HIR
E. CMIR

2. a 20 year-old woman develops disfigure nasal lump 2 month after


recieve bovine collagen filter เซลล์อะไรตอบสนองต่อภำวะนี้
A. Basophil
B. Eosinophil
C. Neutrophil
D. Plasma cell
E. Macrophage
B1.5 Pathogenesis, pathophysiology, basic pathological process and
laboratory investigation
1. A 5 years old boy was hit with a stick at the buttock causing
redness at the overlying skin. Which mediator is contributed to the
redness of the skin?
A. Serotonin
B. Histamine
C. Bradykinin
D. Nitric oxide
E. Thromboxane

2. ผูป้ ่ วยเป็ น HIV diagnosis ว่ำเป็ น Penicillium manefeii จะเจอลักษณะอย่ำงไรใน


laboratory
A. Budding yeast
B. Binary fission
C. Septate hyphae
D.
E.

3. ผูป้ ่ วย cirrhosis ถูกหอยในทะเลบำด gram stain เจอ g-ve bacilli มีโอกำสเป็ นเชื้อใดมำกที่สุด
A. E. coli
B. K. pneumoniae
C. V. vulnificus
D.
E.

4. ผูป้ ่ วยเป็ นSLE กินยำ steroid มำนำน เป็ น pneumonia ตรวจ sputum พบ nematode
larva ผูป้ ่ วยติดเชื้ออะไร
A. Trichinella
B. Ascaris
C.
D. Strongyriodes
E.
F.
5. Japanese tourism complaint for gut obstruction. Physician do
colonoscopy in small intestine and see nematode. the patient tell
about history of raw marine fish consumption.
A. Ascariasis
B. Anisakiasis
C. Trichuriasis
D. Capillariasis
E. Strongyloidiasis

6. ชำยอำยุ 40 ปี ชอบทำนเนื้อวัวดิบบ่อยๆ มีโอกำสติดอะไร


A. Taeniasis
B. Schistosomiasis
C. Fasciolopsiasis
D. Gnathostomiasis
E.

7. A patient has watery diarrhea and rice watery stool. What is the
second messenger mediated in the response?
A.
B.
C.
D. cAMP
E.

8. เด็กมี Koplik spot เกิดจำกกำรติดต่อกันผ่ำนทำงใด


A. Droplet transmission
B. Fecal-oral transmission
C. Vertical transmission
D.
E.

9. Pregnant women, IgM rubella positive กำรตรวจ Ab ควรเก็บสิ่ งส่งตรวจอย่ำงไร


A. Serum room temperature
B. Serum -20c
C. Plasma room temperature
D. Plasma 4 C
E. Whole blood 4 C
10. หญิงคนหนึ่งจะตรวจ Cervical screening ใช้specific
fixative ใด for cytology
A. 70% alcohol
B. 95% alcohol
C. 10% formalin
D. 40% formalin
E. 0.9% sodium chloride

11. ชำย 50ปี มีอำกำรปวดปวดเข่ำ บวม เจำะ synovial fluid อะไรคือ


firstly investigation
A. Mucin clot test
B. WBC count
C.
D. Dry gram stain
E. Wet preparation

B1.6 Gender, ethic and behavioral considerations affecting disease


treatment and prevention, including psychosocial cultural, occupational,
and environment
1. เด็กมี worsen school performance จำกติดเกม สถำบันใดสำคัญที่สุด
A. Family
B. Politic
C. Religion
D. Economic
E. Education

2. A 50 years-old man has well-controlled hypertension for 10 years.


He wanted his doctor to sign him a medical certificate supporting
his early retirement plan. Should the doctor do as he was
requested?
A. Yes, patient's autonomy
B. Yes, patient's beneficence
C. Yes patient's non-maleficence
D. No, doctor's misconduct
E. No, conflict of interest
3. A man was told by his doctor that he has stage 4 colon cancer. He
did not believe his doctor and ignored what he was told. What is
the man's coping mechanism?
A. Denial
B.
C.
D. Regression
E. Suppression

4. Women weight 59 height 155 ถำมว่ำผูห้ ญิงดังกล่ำวมีลกั ษณะใด


A. Unobesity
B. Healthy
C. 1st degree obesity
D. 2nd degree obesity
E. 3rd degree obesity

B1.7 Multisystem process

1. ฝำแฝด A และ Bอำยุ 20 ปี monocygotic twin มีลกั ษณะทำงกำยภำพเหมือนกันทุกประกำร นำ


แฝดคู่ไปตรวจวัดกำรใช้ออกซิเจนด้วย dynamic exercise ที่HR 120 ครั้ง/min พบว่ำ A และ
B 15 , 30 L/kg weight/min ปัจจัยใดทำให้กำรใช้ออกซิ เจนของแฝดคู่น้ ีต่ำงกัน
A. Maximum heart rate
B. Muscle mitochondria density
C. Tissue oxygen extraction ability
D. Stroke volume ที่supply ตอนออกกำลัง
E. Sympathetic activity

2. ผูป้ ่ วยมีภำวะ Hyponatremia decrease urine output increase urine


osmolarity ถำมว่ำเกิดจำกอะไร
A. Angiotensin
B. Aldosterone
C. ADH
D. Renin
E.

3. หญิงอำยุ 40 ปี เป็ น Lupus Nephritis ทำนยำ Prednisolone 40 mg ติดต่อนำน 6 เดือน


มีอำกำร muscle weakness เกิดจำกควำมผิดปกติอะไร
A. Sodium ต่ำ
B. Potassium ต่ำ
C. Calcium สูง
D. Magnesium สูง
E. Phosphate ต่ำ

4. ชำยดมกำวแล้วมีอำกำร muscle weakness ตรวจ muscle power grade 3/5 NaCl


Hco3 K ปกติหมด มีแค่ K ต่ำ
A.
B.
C.
D.
E. Hypokalemic periodic

B1.8 General pharmacology

1. ผูป้ ่ วยชำย HIV ได้รับยำ dideoxynucleoside drug ถำม mechanism ของยำ


A. Abberant splicing
B. Viral mutation
C. Shorten telomere
D.
E. Nucleoside terminate

2. กิน warfarin แล้วเป็ น leukorrhea กิน ketoconazole แล้วมี gum bleeding ถำมว่ำ
ketoconazole ทำอะไรให้เกิด bleeding
A. แย่งจับ albumin
B. เพิ่ม warfarin absorption
C. ลด renal secretion
D. ลด vitamin K absorption
E. ลด biotransformation ที่ตบั

3. ยำกำจัดที่ไต 80% ถ้ำไตพัง 50% จะต้องปรับ maintaining dose เท่ำไร


A. 20%
B. 40%
C. 60%
D. 80%
E. 30%

4. 15 years old girl has cellulitis. Doctor prescribe dicloxacillin.

Why have to eat dicloxacillin after meal?"

A. Increase drug potency


B. Reduce adverse drug effect
C. Increase drug absorption
D.
E.

5. A female patient presents with severe throbbing unilateral


headache. She complains of irritation upon seeing bright light.
Which of the following drugs is appropriate for this patient's
condition?
A. Beta blocker
B. ACE inhibitor
C. Serotonin agonist
D. Nitric oxide donor
E. Calcium channel blocker

6. male 15 y/o DM type1 polyuria ,polydipsia, dypnea pFBS 400 Low


BP high HR,RR
A. Dopamine iv.
B. Dobutamine iv.
C. Dextram solution iv.
D. Manitol iv.
E. NNS iv.

7. กินเหล้ำเถื่อนแล้วเกิดตำเบลอๆ จะแก้ดว้ ยอะไร


A. Ethanol
B. Thiamine
C.
D.
E. Ethylene glycol
8. ยำตัวหนึ่ง มี Vd =0.5 L/kg bioavailability = 30percent ต้องกำรให้ยำถึงควำมเข้มข้นใน
พลำสมำเท่ำกับ 10 mg/L ในผูป้ ่ วยน้ ำหนัก 60 kg ต้องให้ Loading dose เท่ำไร
A. 90 mg
B. 100 mg
C. 300 mg
D. 600 mg
E. 1000 mg

9. ผูช้ ำย 65 ปี เป็ น pyelonephritis ได้รับยำ gentamicin 7mg/day/kg ต้อง monitor


อะไร
A. BUN/Cr
B. AST/ALT
C. FBS
D.
E.

10. ผูห้ ญิงอำยุ 50 ปี ได้รับยำ Glipizide OD แบบ modifying releasing หมอควรให้


คำแนะนำอย่ำงไรในกำรกินยำ
A. กินกับน้ ำอุ่น
B. เอำไปละลำยน้ ำ
C. ห้ำมกัด ห้ำมเคี้ยว
D. สำมำรถลดยำได้ครึ่ งหนึ่ง เมื่อมีอำกำร hypoglycemic
E. เมื่อลืมกินยำ ควรกินยำทันทีที่สุดเท่ำที่ทำได้

B1.9 Quantitative methods

1. In a study of diabetic neuropathy by drug x, 5 out of 100 patient in


treatment group while 10 of 100 patient in control group ตอบสนองต่อ
ผลกำรรักษำ

How many patient should be treat with drug x จึงจะให้ผลกำรรักษำ (ถำมNNT)"

A. 5
B. 10
C. 15
D. 20
E. 40
2. ต้องกำรศึกษำ risk factor ของ zoster. 80 คน เป็ น zoster 230 คนไม่เป็ น zoster จะหำว่ำ
มีใครเคยมีประวัติเป็ น chicken pox
A. Clinical trial
B. Retrospective cohort
C. Case control
D.
E.

3. ทดสอบยำ antihypertensive ปรำกฏว่ำเพิ่ม CVS disease แม้เลือกทดลอง ณ same


age, sex, subject คัดเลือกจำก population เดียวกัน เป็ นเพรำะอะไร
A. Confounder
B. Placebo effect
C. Selection bias effect
D.
E.

4. ศัลยแพทย์ทำกำรผ่ำตัด แล้วไม่จ่ำยยำให้คนไข้ เพรำะงำนวิจยั ทัว่ โลกระบุวำ่ ไม่มีควำมแตกต่ำงระหว่ำงกลุ่มที่


ได้รับยำกับกลุ่มที่ไม่ได้รับยำ
A. Low evidence
B. Moderate evidence
C. High evidence
D. Expert con...
E.

5. ครี มกันแดดยีห่ อ้ a และ b เปรี ยบเทียบประสิ ทธิภำพ โดยให้กลุม่ ตัวอย่ำงทำครี ม a ที่แขนข้ำงหนึ่ง และทำครี ม
b ที่แขนอีกข้ำงหนึ่ง แสดงผลโดยวิธีใด
A. t- test
B. pair t-test
C.
D.
E.

6. ให้ตำรำงมำ แสดงค่ำ odd ratio กับ จำนวนกำรสูบบุหรี่ (มวน/วัน) จำนวนเพิ่ม ค่ำเพิ่ม


A. Dose-response

B2 Hematopoietic and Lymphoreticular System


1. 40 year- old women with chronic liver failure has acute appendicitis
and needs surgical operation. Lab: Hb 7 WBC 14000 ( N 90 L 10)
platelet 300000 Creatinine 8
A. Platelet dysfunction
B. Factor XIII deficiency
C. Primary fibrinolysis
D.
E. Acquired coagulation inhibitor

2. เด็กชำยอำยุ 10 ปี มีอำกำรซีด mild episodic fever mild jaundice PE : anemia


jaundice no hepatospleenomegaly blood smear ดังรู ป
A. Ham test
B. G6PD test
C. Fragility test
D. Direct antiglobulin test
E.

3. 35-year-old man presents with chronic anemia and mild jaundice;


dark urine in the morning is occasionally found.

CBC: Hb 8 g/dL WBC 3,800/cu.mm. Platelet. 90,000/cu.mm.

red cell morphology : normochromic normocytic , polychromasia


few
What is primary defect of red blood cell in this patient?"

A. heme
B. Globin
C. Spectrin
D. G6PD
E. Glycosylphosphatidylinositol anchored protein

4. 70 years old patient have nauseaand vomiting


PE: pallor and swelling both of legs normal MCV , hematocrit 24%,
increas BUN and creatinine
Which of the following associate with cause of anemia in this
patient?"
A. Ferritin
B. Angiotensin
C. Erythropoietin
D.
E.

5. Hb ต่ำ platelet ต่ำ white count ต่ำ normochromic normocytic เกิดจำกอะไร


A. Heme
B. Globin
C. Spectrum
D. G6PD
E. Anchoring protein

B3 Central and Peripheral Nervous System


1. cervical injury pin prick test ที่กน้ + ถำมว่ำอะไรเสี ยหรื อปกติ
A. Corticospinal tract
B. Spinothalamic tract
C. Fasiculus gracilis
D. Fasiculus cuneatus
E.

2. A 30-year-old man has a headache after eating land snails. CSF


contains high eosinophil. What is infecting the patient?
A. Taenia solium
B. Angiostrongylus cantonensis
C. Gnathostoma spinigerum
D. Trichinella spiralis
E. Capillaria philippinensis

3. 30 -year-old man was cut by a sharp knife at his wrist result in a


complete tear of Median Nerve. A surgery was performed. What cell
is responsible for repairing the nerve in the healing process?
A. Neuron
B. Microglia
C.
D.
E. Schwann cell
4. หญิงอำยุ 50 ปี resting tremor , decreased facial expression , short-step
gait มีพยำธิสภำพบริ เวณใด
A. Flocculonodular lobe
B. Substantia nigra
C. Red nucleus
D. Frontal lobe
E. Putamen

5. ผูห้ ญิงอำยุ 45 ปี มีควำมคิดจะฆ่ำตัวตำย neurotransmitter ตัวใดจะมีค่ำต่ำกว่ำปกติ


A. Serotonin
B. Glutamate
C. Dopamine
D. Acetylcholine
E. Gamma-aminobutyric acid

6. เลิกบุหรี่ ใช้ยำอะไร
A. Bupropion

7. เลิกเหล้ำทันที ต้องระวังอำกำรใดใน 6 เดือนแรก


A. Tremor

B4 Skin and Related Connective Tissue


1. Bilateral groin itchy lesion clear cental clear border HIV positive
A. Versicolor
B. Tinea cruris
C.
D.
E.

2. ทหำรฝึ กอยูก่ ลำงแดดอุณหภูมิ 40องศำต่อมำมีอำกำร fainting และ unconcious เป็ นอะไร


A. Heat exhaust
B. Heat syncope
C. Classical heat stroke
D. Exertional heat stroke
E. Cerebrovascular stroke
3. A 2-year-old girl has dermatitis. She had a topical corticosteroid.
Which of the following medication is the best choice for the girl?
A. Hydrocortisone
B. Prednisolone
C. Betamethasone
D. Triamcinolone
E. Desoximetasone

B5 Musculoskeletal System
5. Loss of sensation at dorsal of foot weaknees dorsiflexion and
eversion โดรที่ nerve ใด
A. Tibial
B. Peroneal
C. Sciatic
D.
E.

6. A 7-year-old boy with head injury from falling from high chair.
Physical examination shows periorbital and nasal bruising, CSF
rhinorrhea. Which of the following structure is injured?
A. Clivus
B. Cribiform plate
C. body of sphenoid bone
D. basilar part of occipital bone
E. squamous part of temporal bone

7. 15 years old man painตั้ง left kneeลงมำ intramedullary mass


metaphysis hypochromatic spindle
A. Osteosarcoma
B. Chrondrosarcoma
C. Osteoblastoma
D. Osteochrondoma
E.

8. A 30 years old female presents with recurrent muscle weakness.


She also experiences dysphagia and diplopia. Upon examination,
he develops ptosis on both eyes after prolonged blinking. What is
the most likely finding in this patie "
A. Anti-tubulin
B. Anti-GM1 ganglioside
C. Anti-galactocerebroside
D. Anti-central myelin antigen
E. Anti-acetylcholine receptor

9. ผูช้ ำยอำยุ50ปี เป็ น gout กินยำ colchicine ต้องระวัง side effect อะไร
A. Dizziness
B. Diarrhea
C. Vomiting
D. Rash
E.

B6 Respiratory System
1. Acute asthma ใช้ยำไร
A. Montelukast
B. Prednisolone
C.
D.
E.

2. A 1-year-old boy has severe cough in cluster, more during the


night, for two weeks. He has not received any vaccines.

PE: subconjunctival hemorrhage, normal breath sound.

What is mechanism of this condition?"

A. Mucosal inflammation
B. Change of normal flora
C. Increased mucus secretion
D. Pseudomembranous formation
E. Reduce mucociliary clearance

3. Male 70 years-old มำด้วยอำกำร dyspnea & productive cough นัง่ lean ไป


ด้ำนหน้ำ และทำ purse lip breathing
PE: barrel - shaped chest ถำมpathogenesis ของโรค
A. Increase TGF-B
B. Excessive endothelin1
C. Mutation of CFTR gene
D. Type1 hypersensitivity
E. Protease and antiprotease imbalance

4. หญิงอำยุ40 เป็ นไข้สูง dyspnea วินิจฉัยเป็ น pneumonia. เชื้อก่อโรคที่น่ำจะเป็ นมำกสุดคือเชื้อใด


A. S. aureus
B. Chlamydia
C. H. influenza
D. Mycoplasma pneumoniae
E. S. pneumomiae

B7 Cardiovascular System
1. Heavy smoker ปวดขำเวลำเดินไปได้นิดหน่อยก็เจ็บขำ ปลำยนิ้วมือนิ้วเท้ำเย็น diagnosis
A. Scleroderma
B. Polyarteritis nodusa
C. Thromboangiitis obliteran
D. Takayasu arteritis
E.

2. A 40 year-old man has had (อำกำร angina) ถำม EKG


A. A
B. PR
C. QRS
D. QT
E. T

3. A 30-year-old woman is diagnosed with pulmonary valve stenosis.


Which of these are the best mean axis deviation?
A. A.-100 to -50 degrees
B. -50 to 0 degrees
C. 0 to +50 degrees
D. +50 to +100 degrees
E. +100 to +150 degrees"
4. ถ้ำรับประทำนยำ nitrate เกินขนำด ต้องระหว่ำง S/E ใด
A. Hypotension

B8 Gastrointestinal System
1. superficial inguinal hernia อยู่ medial ต่อโครงสร้ำงอะไร
A. Lateral umbilical ligament
B. Inguinal ligament
C. Urachus
D. Spermatic cord
E. Rectus abdominis

2. A patient has tenemus due to inflammation of inferior part of the


anal canal below the pectinate line that spread into ischioanal .
which the following lymph node is swollen first ?
A. Internal iliac
B. External iliac
C. Common iliac
D. Superficial inguinal
E. Deep inguinal

3. ผ่ำตัด Large area of sigmoid colonต้องผูกเส้นเลือดใด


A. Superior rectal a.
B. Superior mesenteric a.
C. Inferior mesenteric a.
D.
E.

4. จะรู ้ได้อย่ำงไรว่ำผูป้ ่ วยมีภำวะ portal hypertension


A. "
B.
C.
D.
E. Esophageal varice

5. A 45 year-old man presents with dizziness and confusion. PE :


jaundice, spider nevi, ascites. Laboratory findings : total protein 4.5(
ต่ำ), albumin 1.8 total billirubin 14.8, direct bilirubin 8 AST
103,ALT 47,ALP 55, ammonia 250
what is the most likely cause of these clinical presentations?"

A. Alcoholic cirrhosis
B. Intracranial lesion
C. Fulminant hepatitis
D. Hepatic encephalopathy
E. Wernicke encephalopathy

6. ผูป้ ่ วยมีอำกำร severe epigastric pain serum amylase lipase สูง ลักษณะใดต่อไปนี้
แสดงถึง poor prognosis ที่สุด
A. Hypocalcemia
B. Hypertriglyceridemia
C. Increase sweat bicarbonate
D. Urease breath test positive
E. HBsAg positive

7. A 55 year old woman weight 80kg height 155 cm .she has a history
of hypertension what is the risk
A. Peptic ulcer
B. Diverticulosis
C. Osteoporosis
D. Endometrial carcinoma
E. Collagen vascular disease

8. ติดเหล้ำ เป็ น cirrhosis จนเกิดportal-systemic encephalopathy ยำใดช่วยลด


encephalopathy ที่เกิดได้
A. Lactulose
B. Ranitidine
C. Sucralfate
D. Loperamide
E. Omeprazole

B9 Renal/Urinary System

1. 40 years old man, cushing syndrome with hyperkalemia. Which cell


plays important role?
A. Glandular cell
B. Mesangial cell
C. Principle cell
D. Intercalated cell
E.

2. เพรำะเหตุใด จึงมีปัสสำวะออกมำเพียงครึ่ งหนึ่งของน้ ำที่กินไปตอนก่อนเข้ำนอน


A. Increase ADH
B. Increase Renin
C. Increase epinephrine
D. Decrease aldosterone
E. Decrease angiotensin II

3. 65-year-old man with skin ulcer ใช้ cloxacillin a few day ago มำด้วย
fever, rash, oliguria urinalysis: RBC 3-5/hpf WBC 10-20/hpf มี renal
function impairment ข้อใดต่อไปนี้จะพบใน renal biopsy
A. Plasma cell tubulitis
B. Neutrophilic tubulitis
C. Granulomatous vasculitis
D. Interstitial neutrophilic infiltration
E. Interstitial eosinophilic infiltration

4. A 10-year-old girl present with headache ,puffy eye lid, bilateral leg
edema for 2 day. Two year ago she had sore throat, fever relived
with over the counter drug ถำมว่ำเป็ นอะไร

UA : protien2+, rbc 5-10 , rbc cast 1-2 ถำมว่ำเป็ นอะไร"

A. SLE
B. Acute kidney injury
C. Minimal change disease
D. Drug induce interstitial nephritis
E. Post streptococcal glomerulonephritis

5. ผูป้ ่ วยมีควำมดันโลหิ ตสูงและเป็ น PTH secreting CA มี Ca สูง ต้องให้ยำ/fที่เหมำะสมเพื่อลดควำม


ดันโลหิ ตสูงในผูป้ ่ วยรำยนี้
A. Furosimide
B. HCTZ
C. Acetazolamide
D. Mannitol
E. Spironolactone

6. ชำยอำยุ 20ปี กินprotein ทำไมฉี่ เยอะ (ภำยใน4hr)


A. Increase plasma urea
B. Increase plasma tonicity
C. Increase plasma Na concentration
D. Decrease plasma renin
E. Decrease renal Na reabsorption

7. A boy has urine leaking from his umbillicus. What is likely to be


found?
A.
B.
C.
D. Patent vitelline duct
E. Patent allantois
B10 Reproductive System
1. Pudendal n block ที่ labour structure บริ เวณใดจะไม่เสี ย
A. Buttock
B. Uterus
C. Mons pubis
D. External sphincter
E. Urinary bladder

2. A 60 years old multiparous woman presents with a mass


protruding from the vagina. Uterine prolapse is diagnosed. Which
structure is weak and responsible for this condition?
A. Perineal body
B. Broad ligament of uterus
C. Round ligament of uterus
D. Transverse perineal muscle
E. Transverse cervical ligament

3. 8 years old boy height 97 percentile, enlarge penis small testis


musculine and cracked voice . Which hormone is high
A. Corticosterone
B. 17-hydroxyprogesterone
C. 11-deoxycorticosterone
D. CRH
E. Somatotroph hormone

4. 40 years old housewife has recurrent fungal vaginitis. she has 2


children. Her husband is construction worker. cause?
A. Malnutrition
B. Low economy
C. Poor hygiene
D. Antifungal drug resistant
E. Reinfection from husband

5. Woman 25 years มีลกู หลัง่ น้ ำนมมำ 6เดือน พบว่ำที่ผำ่ นมำamenorrhea ถำมว่ำ


mechanism ใดทำให้เกิดภำวะดังกล่ำว
A. Stress
B. GnRH ต่ำลง
C. Oxytocin ต่ำลง
D. Dopamine ต่ำลง
E. Prolactin ต่ำลง

6. ผูห้ ญิงอำยุ 35 ปี มำด้วยอำกำรอ่อนแรง มีประวัติ postpartum hemorrhage พบ lactating


amenorrhea ถำมว่ำพยำธิน่ำจะมำจำกอะไร
A. Anemia
B. Ovarian failure
C. Hypothyroidism
D. Sheehan syndrome
E. Adrenal insufficient

7. 65 years old man presents with bilateral progressive enlargement


of breast. He had been taking herb medicine for roughly one year.

What is the most likely component of the herb medicine ?"

A. Progesterone
B. Estrogen
C. Testosterone
D. DHEA
E. Androgen

8. ผูห้ ญิง 30ปี เป็ น severe dysmenorrhea พบ nodule และ cystic ที่right adnexa
A. Metastasis adenocarcinoma
B. Small capillary proliferation
C. Endometrium gland and stroma
D.
E.

9. ผูห้ ญิง พบว่ำมีกอ้ นที่ groin ตรวจ โครโมโซมพบ 46 XY ถำมว่ำเป็ นอะไร


A. Testis dysgenesis
B.
C.
D. 5 alpha reductase deficiency
E. Complete androgen insensitivity

10. Men milk like discharge from nipple เกิดจำกอะไร


A. High GnRH
B. High FSH
C. High estrogen
D. Da antagonist
E.

11. ภำวะPlacenta previa blastocyst จะไปฝังตัวที่ตำแหน่งใด


A.
B.
C.
D.
E. Internal os of uterus

12. ชำย มำด้วย อำกำรปั สสำวะยำก มี dribbling PE : smooth firm enlarge


prostate gland ตรวจ PSA 3 (0-4) ได้รับยำถำมว่ำยำออกกลไกอย่ำงไร
A. aromatase inhibitor
B. 5-alpha-reductase inhibitor
C.
D.
E.
13. ผูห้ ญิงอำยุ 30 ปี ไม่มีประจำเดือนมำ 2 เดือน มีประจำเดือนวันสุดท้ำยวันที่ 10 มกรำคม 2015 ถ้ำผูห้ ญิง
คนนี้ต้งั ครรภ์ จะมีกำรฝังตัวของตัวอ่อนในวันใด
A. 17 มกรำคม 2015
B. 24 มกรำคม 2015
C. 31 มกรำคม 2015
D. 7 กุมภำพันธ์ 2015
E. 14 กุมภำพันธ์ 2015

B11 Endocrine System

1. A 20 year-old woman have anterior neck mass. Blood test show


free T4 &T3 within normal limit, high TSH, what is the most likely
diagnosis?
A. Multinodular goiter
B. Secondary hyperthyroidism
C. Secondary hypothyroidism
D. Subclinical hypothyroidism
E. Subclinical hyperthyroidism"

2. คนไข้เป็ นDM แนะนำให้ออกกำลังกำยถำมว่ำออกกำลังกำยช่วยอะไร


A. Insulin sensitivity

3. 65-year-old woman confusing for 2 hours เคยใช้ steroid containing


herbal pill for 3 years ใช้เพื่อ relieve her knee pain แต่หยุดใช้ยำเมื่อ 2 วันก่อน
PE: BP 70/40 HR 120/min ข้อใดจะลดลงใน case นี้
A. Sympathetic activity
B. Plasma angiotensin II
C. Plasma potassium
D. Aquaporin 2 expression in collecting duct
E. Arterial alpha adrenergic receptor expression

4. 50 year old woman comes with dull headache. BP 150/90. HR 100.


She has left suprarenal mass. Vanillylmandelic acid test positive in
urine. What is her diagnosis?
A. Metastasis cancer
B. Pheochromocytoma
C. ACTH secreting tumor
D. Secondary aldosterone tumor
E. Adrenal cortical carcinoma

5. woman eating high protein low carbohydrate food for diet.

Which hormone is most likely secrete due to her meal ?"

A. Cortisol
B. Secretin
C. Glucagon
D. Norepinephrine
E. CCK

6. หญิงอำยุ30ปี บ่นว่ำน้ ำหนักขึ้น 5kg ใน2เดือน พบก้อนที่ดำ้ นซ้ำย ขนำด3.5cm PE:PP 110 BP
140/90 no bruise T3 230 T4 2.3 TSH<0.01 จ งdiagnosis
A. Thyroiditis
B. Simple goiter
C. Thyroid cancer
D. Toxic adenoma
E. Grave's disease

7. กลไกของ metformin
A. Bind PPAR gamma
B. Decrease hepatic gluconeogenesis
C.
D.
1. ยำรักษำต่อมลูกหมำก ถำม side effect : hypotension
2. side effect ของกำรรักษำ gout ด้วย cochicine : diarrhea
3. Danazol : androgen agonist
4. Hypotensive drug ที่ทำให้เกิดหน้ำแดง ตำตุ่มบวม : Amlodipine

5. Cyclospora
6. ยำเลิกบุหรี่ : Bupropion
7. Distant heart sound + neck vein engored ให้ ECG มำ : cardiac
teamponade
8. Distilling home liquid antidote : ethanol
9. ผมร่ วงศีรษะไม่มีแผล ผมยำวขนำดต่ำงๆกัน : mania
10. Vulva carcinoma drain lymph node อันไหน : superficial inguinal
Lymph node
11. ปั สสำวะออกมำเป็ นอุจจำระ : urorectal septum
12. หำยใจเข้ำแบบรุ นแรง : sternocleidomastoid
13. เป่ ำ(หำยใจออกอย่ำงรุ นแรง) : abdominal muscle + internal intercostal
muscle

14. Burkitt lymphoma : EBV


15. ปั สสำวะออกมำทำงสำยสะดือ : allantois

16. อะไรพัง : urethral fold

17. Pneumocystis jiroveci


18. ส่งเลือดที่จะตรวจ Glucose ต้องใส่หลอดที่มีสำรป้ องกันเลือดแข็งอะไร : fluoride
19. กลไกของ atropine : antimuscarinic receptor
20. กิน atropine รักษำ organophosphate ยังเหลืออำกำรอะไรอยู่ : muscle
fasciculation
21. ยำอะไรห้ำมกินคู่กบั แคลเซียม : fluoroquinolone (…..ofloxacin)
22. Heparin ต้อง monitor อะไร : aPTT
23. ยำอะไร side effect สี สม้ : rifampin
24. กลไก psoriasis : Th1 , Th17
25. กลไก metformin : ลด hepatic gluconeogenesis
26. Vancomycin : cell wall
27. เลิกเหล้ำ 6 ชัว่ โมงเกิดอะไร : tremor
28. รักษำ hypercalcemia : furosemide
29. ถ้ำงูทะเลกัด ให้ระวังอะไร : hyperkalemia
30. รด.ฝึ กทหำร : Exertional heat stroke
31. ตัด thyroid gland แล้วให้ tousseu sign ถำมว่ำจะพบอะไร :
hyperphosphatemia
32. digeorge syndrome จะไม่มีอะไร : parathyroid hormone
33. เด็กไอเสี ยงก้อง ไม่มีเสี ยงแหบ : trachea
34. นอน แล้วเลือดออกในร่ ำงกำยจะไปกองที่ : hepatorenal recess
35. คนท้องรับควันบุหรี่ : ลดperfusion
36. Cachexia อ่อนแอ น้ ำหนักลด : TNF alpha
37. Strawberry tongue
38. ผูห้ ญิงคลอดลูก เสี ยเลือดเยอะ น้ ำนมไม่ไหล เมนส์ไม่มำ : Shebaan syndrome
39. Tendon xanthoma : LDL
40. ชำยอ้วน ภัตตำคำรจีน : เสี่ ยง CAD
41. ให้อำกำรของ angina pectoris มำ แล้วถำม EKG

42.
43. Nerve พัง อะไรซ่อม : Schwann cell
44. ให้ chemotherapy หลำยวัน ยังทีทอ้ งเสี ยมีเลือด : C. diff
45. Nasal necrosis , non septal hyphae : Rhizopus
46. Pap smear : 95% alcohol
47. ไข้, RBC เปลี่ยนเป็ นรู ป bite cell : G-6-PD def
48. ให้อำกำร RA : FAB IgM
49. TSH สูง , T3 ปกติ , T4 ปกติ : subclinical hypothyroid
50. Recurrent respiratory infection infertile : microtubule พัง
51. Mycotic aneurysm : infective endocarditis
52. Pudendal nerve ใช้อะไรเป็ น landmark : ischial spine
53. Ankle sprain medial malleolus : deltoid ligament
54. Sigmoid colon : inferior mesenteric artery
55. หัด ติดทำงไหน : air droplet
56. กินเหล้ำ ทำไม uric เยอะ : ลดกำรขับออก
57. หลัง่ กรดในกระเพำะ : parietal cell
58. ยำ GERD : omeprazole
59. CPR เริ่ มโดย chest compression
60. Albinism ตรวจ enzyme อะไร
61. Purpura ตรวจ lab อะไร
62. anorectal wart ใช้ยำอะไร
63. ยำเหน็บ ulceritis colitis , crohn’s disease
64. ให้อำกำรของ Wernicke มำ ถำม area ไหนพัง
65. ไข่ลงภุงแค่ขำ้ งเดียว ขำด Hormmone อะไร
66. Neutrophil diapedesis ใช้ยำอะไร
67. ลักษณะทำง patho ของ CA breast type 1
68. ภรรยำนอนกรน มือใหญ่ เสี ยงใหญ่ข้ ึน : acromegaly
69. เมำแล้วเปิ ดบ้ำนได้เอง เกิดจำก tract อะไร

1. Chochicine the most concern ADR

- Rash,Diarrhea

2. ได้ PTU มีอำกำร อำกำรไข้ไอ ควรตรวจอะไร

- CBC (น่ำจะเป็ น agranulocytosis ซึ่งเป็ นผลข้ำงเคียงที่อนั ตรำย ควรนัดตรวจ CBC เรื่ อยๆหรื อเมื่อมี
อำกำรคล้ำยๆติดเชื้อ)
3. มี black urine ตอนเช้ำ ซีดเป็ นช่วงๆ ……….

- GPI anchors protein (น่ำจะเป็ น PNH )

4. มีรูป

- Burkitt lymphoma (starry sky)

5. HIV ท้องเสี ยเป็ นน้ ำ stool exam ย้อม


AFB

-cyclospora cayetanensis

6. เป็ นปอดบวม….. ให้ MAHA blood picture มำ ถำมว่ำผลที่ตรวจได้ควรจะเป็ นอย่ำงไร

- Free haptoglobin ต่ำลง (เนื่องจำกเป็ น intravascular hemolysis)


7. - hereditary spherocytosis

8. ให้ตวั บวม protein 3+ ฉี่มี fomy RBC 50-100 พยำธิสภำพเป็ นแบบใด

- minimal change

- MN

- FSGN

- อะไรซักอย่ำงที่เป็ น nephi

9. recurrent UTI . Pelvis ureter บวม

- ureteropelvic influx

10. หญิงไทยอำยุ 35 ปี ลดควำมอ้วนด้วยกำร งดอำหำรประเภทคำร์โบไฮเดรต และทำนอำกำรประเภทโปรตีนแทน


ฮอร์โมนใดจะหลัง่ เพิ่มขึ้นมำกที่สุด

- secretin

-cck

-glucagon

-NE

-cortisol

11. เป็ น PKU ห้ำมกิน aspatam(สำรให้ควำมหวำนแทนน้ ำตำล)


12.

- pneumocystis jiroveci

13. Antidoteของmethanol.

-ethanol

14. Rifampin ADR

- ฉี่ สีสม้ .

15. มีcsf rhinitis จำกโดนรถชนกระดูกอะไรมีปัญหำ

- มีชอ้ ยcribifirom

16. ปวดกล้ำมเนื้อไปว่ำยน้ ำโดนสัตว์กดั

- งูทะเลเกิดhyperk

17. Hyperkเห้นในekgยังไง

- tall peak T

18. คำนวน EF

- =sv/edv

19. โดนshellfish.ตำ

- vibrio ..
20. Drowning เกิดอะไรในปอด

-เพิ่มdead space -ลดalveolar recoil

21. ใช้อะไรรักษำhypercalcemiaแบบชัว่ ครำว

-furosemide

22.รักษำไมเกรนตอนacute

23. อะไรมี psammoma bodies

- papillaCA thyroid

ภำคเช้ำ

48. ผูป้ ่ วยชำย น้ ำหนักขึ้น central obesity , proximal muscle weakness, abdominal
striae ,blood glucose 200 mg/dl ถำมว่ำใช้ investigation ใดในกำร specific
diagnosis
A. Lipid profile
B. Thyroid function test
C. Glucose tolerance test
D. 24 hr. urine VMA
E. Dexamethasone suppression test
หญิง 60 ปี มำด้วยอำกำร chronic hip pain แพทย์ได้ทำกำรผ่ำตัดเปลี่ยน hip แล้ว และได้ตดั ชิ้นเนื้อตรง
head of femur ส่ง pathologist ผลออกมำเป็ น wedge shape area with pale.What
is the most diagnosis? a.avascular necrosis b.metastasis cancer
c.mycobacterial infection d.deficiency of mineral e.

99. หญิงตั้งครรภ์ ตรวจ antibody ต่อ rubella แต่ตรวจไม่ได้ทนั ที ต้องเก็บไว้ 72 ชัว่ โมง ต้องเก็บยังไง
1. Serum -20 องศำ
2. Serum room temp
3. Plasma 4 องศำ
4. Plasma room temp
5. Whole blood 4 องศำ
100. หญิงเป็ นแม่บำ้ น ติด recurrent candida vaginitis แม้จะทำกำรรักษำ สำมีทำงำนก่อสร้ำง มีลูก 2
คน สำเหตุคือ
1. Malnutrition
2. Low sociaeconomic status
3. Poor personal hygiene
4. ติดจำกสำมี

99. Lung cancer แล้วมีอำกำร Hyponatremia ฉี่ นอ้ ยและ osmolality สูง เกิดจำกฮอร์โมนใด
1. ADH
2. Aldosterone
3. ACTH

100. เป็ น papillary thyroid carcinoma แล้วทำ thyroidectomy จำกนั้นเกิด numbness,


tetany PE: Trousseau sign positive เกิดจำกอะไร
1. Hypercalcemia
2. Hypovitaminosis D
3. Hyperphosphatemia
4. Hypermagnesemia

98. หญิงสำวอำยุ 20 ปี ถูกหมำข้ำงถนนกัด หลังจำกต้องทำแผลแล้ว ก็ตอ้ งฉี ด Rabies vaccine ทันที และอีก
3, 7, 15, 30 วันข้ำงหน้ำ รวมทั้งหมด 5 ครั้ง ทำไมต้องฉีดวัคซีนหลำยรอบ และห่ำงกันโดยมี interval เท่ำนี้
A. To increase IgM
B. To increase vaccine immugenicity
C. To stimulate sufficient neutralized antibody
D. To stimulate both innate and adaptive immunity
E. To avoid side effect from single dose vaccination

99. ชำย Leukemia รักษำโดยกำรทำ Bone Marrow transplantation แล้วกินยำกดภูมิ ช่วง 2


สัปดำห์แรกก็มี Blood cell count increase จำกนั้นเมื่อครบ 2 เดือน ได้มี Diarrhea,
Jaundice, Hepatomegaly ถำมว่ำเกิดจำกอะไร
A. Graft versus host disease
B. Leukemia relapse
C. Chronic graft rejection
D.Cyclosporine A toxicity
E.Opportunistic infection

98.กินอำหำร high protein, low carbohydrate ฮอร์โมนใดจะถูกกระตุน้


A. Secretin
B. CCK
C. Epinephrine
D. Cortisol
E. Glucagon
99. Subtotal thyroidectomy ต้องระวัง nerve ใด
A. Vagus
B. Recurrent laryngeal
C. Superficial laryngeal
D. External laryngeal
E. Internal laryngeal
119. Study compare efficacy of anti-diabetic drug ในผูป้ ่ วยทั้งหมด 50 คน
mean Hba1c 95% CI
ยำ A 8.2 7.0-9.8
ยำ B 7.8 6.8-9.3
ถ้ำเรำศึกษำในคนทั้งหมด 200 คน จะเป็ นอย่ำงไร ?
A. Low p-value
B. Low mean Hb1c
C. High mean Hb1c
D. Low power
E. Wided 95% CI

ให้โจทย์มำเป็ น DNA template ให้ ถอดรหัสเป็ น mRNA

64.ผูป้ ่ วยปวดท้องแบบ dullness ที่บริ เวณ umbilical region ถำมว่ำสัมพันธ์กบั อวัยวะใด.


1.parietalperitoneum

2.small bowel

3.gall bladder

4.stomach

65.ข้อใดคืออำกำรของportal hypertension

1.esophageal varice.

XX. during 8 hours sleeping มีกำรผลิตปั สสำวะออกมำเล็กน้อย ทั้งๆที่ดื่มน้ ำครึ่ งลิตรก่อนนอน ถำมว่ำ
อะไรที่ทำให้มีปัสสำวะน้อยตอนกลำงคืน

A.increase renin

B.increase renin

C.decrease angiotensinII
D.decrease aldosterone

124. ผูป้ ่ วยมำด้วย dyspnea from toxic substance.Doctor give adrenerine for
bronchodilator.What is the type of antagonism

A. Chemical

B.Anatomical

C. irreversible

D. physiological

E. Pharmacological

125. ผูป่วยมำด้วย dysuria.So doctor prescribe antimicrobial but pt has Ca


supplement. What antimicrobial was not taken with Ca supplement

A.Co trimoxazone

B. Co amoxiclav

C. athritomycin

D. ceftoxine

E. ofloxacin

79 Autosomal recessive disease has prevalence 1:40000 Which is the


following is the risk of carrier
A. 1/100

B. 1/200

C. 1/400

D. 1/600

E. 1/800

81 อะไรควรเก็บไว้ในroom temp

A.CBC
B.CSF

C.Sputum

D.Urine

E.Blood gas D

XX. ผูป้ ่ วยหญิงมำด้วยอำกำรหนังตำบวมและขำบวมทั้งสองข้ำง


ตรวจโปรตีนในปั สสำวะพบ protein 2+
โจทย์ถำมกลไลที่ทำให้เกิดอำกำรบวมน้ ำ
ตอบ oncotic pressure ลดลง

93.ผูป้ ่ วยรำยหนึ่งไปดำน้ ำที่ทะเลแล้วปวดขำและมีรอยเขี้ยว(fang) ผูป้ ่ วยรำยนี้มีอำกำรแสดงอะไร

1.DIC

2.Compartment syndrome

3.respiratory failure

4.hypokalemia

XX. ชำยอำยุ 40 ปี ติดเชื้อHIVมำ 10 ปี CD4 count 50 cells/mm3 ช่วง 2 เดือนที่ผำ่ นมำถ่ำยอุจจำระ


เป็ นน้ ำวันละ 5-6 ครั้ง กิน norfloxacin แล้วไม่ดีข้ ึน นำ stool มำย้อม modified acid-fast stain
พบจุลชีพขนำด 8-10 ไมครอนดังรู ป จุลชีพใดน่ำจะเป็ นสำเหตุก่อโรคในผูป้ ่ วยรำยนี้
A. Isospora belli
B. Giardia lamblia
C. Toxoplasma gondii
D. Cyclospora cayetanensis
E. Cryptosporidium parvum

140. ทำtestเพื่อDxโรค ในประชำกรกลุ่มหนึ่งที่มีprevalence=10% ถ้ำหำกเปลี่ยนเป็ นประชำกรที่


มีPrevalenceของโรคนี้20% ค่ำใดจะเพิ่มขึ้น

A. Accuracy

B. Sensitivity
C. Specificity

D. Negative predictive value

E. Positive predictive value

141. ผูป้ ่ วยหญิงอำยุ60ปี มำด้วยอ่อนเพลีย พบhypochromic microcytic 1+,MCV70,serum


ferritinต่ำ ถำมว่ำ ควรส่งตรวจใดต่อ

A. Hb typing

B.

C. Coloscopic

D. Abdominal CT scan

E. Bone marrow aspiration

XX. เด็ก 15 ปี เป็ น DM 1 polyuria polydypsia dypsnea temp 36.3 BP 80/50


RR28 PR120 blood glucose 400 ควรtreat ด้วยอะไรก่อน

1. Dobamine

2. Dobutamine

3. Dextran

4. Mannitol

5. Normal saline

XX.ตับแข็ง fibrosis เกิดจำก cell อะไร

1.mac

2.fibroblast

3.statelet cell

XX. ชำยมี incidence of ischemic heart disease มำกกว่ำหญิง 5 เท่ำแต่ prevalence no


significant different จะอธิบำยได้วำ่ อย่ำงไร
A. more risk factor in male
B. higher case mortality rate in male
C. higher 5-year survival rate inn male
D.higher crude morbidity rate in male
E.shorter natural disease in female

120.one outlier is found 20 medical data collected in comunity parameter


affected by outlier ?
a.mode
b.mean
c.median
d.p50
e.interQ range
121. what bacterial component is target of "vancomycin"?
a.cell wall
b.ribosome
c.gyrase
d.cell mb
e.hydropteroate syntase

XX. สุ่มตรวจพบคนไข้ herpes zoster 80คน กับคนปกติ 120คน ถำมว่ำใช้กำรออกแบบกำรศึกษำอะไร


a.case control
b.จำไม่ได้
c.prospective cohort
d.retrospective cohort study
e.จำไม่ได้

XX. ชำยอำยุ 55 ปี เป็ น Chronic kidney disease มีผล Lab พบ K+ สูง ถำม ECG เป็ นอย่ำงไร
ตอบ peak tall T wave

ภำคบ่ำย

C 119. จะทำ pudendal nerve block


What is landmark for injection
A. Ischial spine
B. Ischial ramus
C. Ischial tuberosity
D. Obturator foramen
E. Inferior ramus of pubis

48. 60 years old man had chest pain on exertion relieved by rest. Which
of the following drug mechanism is the most likely to improve this pt.
symptom.
A. Lipoxygenase inhibition
B. Phospholipase A inhibition
C. Increase Nitric oxide production
D. Increase K channel activation
E. Increase Na-K ATPase activation
49. Newborn 10 months old girl passes her meconium through vagina
PE. No anal opening seen further examination show she had fistula
connecting rectum to vagina. What is embryonic membrane is deficit in
this girl.
A. Proctodeum
B. Cloacal membrane
C. Urorectal septum
D. Urogental sinus
E. Distal hindgut

หญิง 65 ปี มำด้วย dyspnea on exertion PE: cardiomegaly, tortous aorta ,


systolic murmur over aortic valve, BP 150/70 mmHg.What is tha cause of
wide pulse pressure?
a.......

b.......

c.decrease aortic compliance

d.increase ejection fraction


e.increase TPR

XX. ผูป้ ่ วย AST 300 ALT 90 ALP ปกติ มำด้วยอำกำรซึม มึนงง สับสน มีอำกำรสั่ น่ ammonia
blood 500 สำเหตุคืออะไร

1.alcoholism
2.hepatic encephalopathy
3.fulminant hepatitis
4.wernike encephalopathy
XX. ชำยวัยรุ่ นคนหนึ่ง มีอบุ ตั ิเหตุทำงรถยนต์ แล้วต่อสำยNG แล้วได้น้ ำสี กำแฟ(coffee ground
appearance) มีerosionในกระเพำะอำหำร แล้วติดว่ำชำยคนนี้เป็ นอะไร

A.peptic ulcer
B.curling ulcer
c.acute gastitis
D.atrophic gastritis
10. ผูป้ ่ วย hip replacement แล้วเกิด trendelenberg gait เกิด injury ที่ nerve ใด

A. Sciatic

B. Peroneal

C. Femoral

D. Sphenous

E. Superior gluteal

11. Preterm neonate เกิดอำกำร tachypnea , subcostal retraction เกิดจำกควำมผิดปกติ


ของ cell ใด

A.-

B. Alveolar cell

C. Endothelial cell

D. Broncheolar cell

E.Alveolar macrophage
XX. ผูป้ ่ วย nullparous found left breast mass --- biopsy :proliferative of
epithelium and stomal and epithelium lining with loose fibroblastic. Dx

A. fibroadenoma

B. galactocele

C. lactational adenoma

XX. -- years old patient come to doctor with lesion on labia majora biopsy
show Squamous cell carcinoma which is the first lymph node that the
cancer metastatis?
A. External iliac

B. Internal iliac

C. para aortic

D. Superficial inguinal

E. Deep inguinal

XX. หญิงวัย30ปี มำพบแพทย์ดว้ ยปั ญหำinfertile แพทย์จึงให้FSHไป ถำมกลไกยำนี้คืออะไร

A.Stimulate ovulation

B.Maturation of many follicles

C.stimulate secondary meiosis of oocyte

D.decrease mucous production from endocervix

E.delay movement of ovum at follopian tube

150.โรค ichthyosis เกิดจำกควำมผิดปกติของโครงสร้ำงใด

1.fibrin

2.keratin

3.desmosome
ชำยอำยุ 40 ปี กินเนื้อวัวสุกๆดิบๆ เป็ นประจำ เสี่ ยงต่อกำรเป็ นโรคใด
A. Taeniasis
B. Sparganosis
C. Cysticercosis
D. Fasciolopsiasis
E. Gnathostomiasis

140. เกิดAsthma attack ใช้muscle ใดช่วยเพิม่ ปริ มำตรthoracic cavity

A. Platisma

B. Subclavius

C. Pectoris major

D. Pectoralis minor

E. Sternocleidomastoid

141. ผูป้ ่ วยดื่อเหล้ำอย่ำงหนักมำหลำยปี หำกเลิกเหล้ำกะทันหัน ใน6ชัว่ โมงแรกต้องระวังอำกำรใด

A. Tremor

B. Runny nose

C. Muscle ache

D. Increase Appetite

E. Profound global confuse

XX. ชำย 40 ปี ทำนยำ carbonic anhydrase inhibitor รักษำ glaucoma ทำให้ urine
hyperbicarbonate ถำมว่ำจะเกิด defect absorp ที่ไหน
A.proximal tubule
B.ascending limb
C.descending limb
D.distal tubule
E.collecting duct

120. ชำย 14 ปี สุขภำพแข็งแรง กินน้ ำ 1 ลิตรภำยใน 15 นำที


อีก 30 นำทีต่อมำ ปั สสำวะมี specific gravity 0.002 กลไกคืออะไร?
a.เพิ่ม ADH
b.ลด filtration fraction
c.เพิ่ม aldosterone
d.ลด GFR
e.ลด aquaporin receptor

คนไข้ชำย สูบบุหรี่ มีอำกำรเดินแล้วปวดน่อง ปลำยมือเท้ำเยน ซีด จงวินิจฉัย a buerger dz


b PAN
c
XX. เด็กอำยุ 10 ปี ต้องผ่ำตัด PDA (patent ductus arteriosus) แพทย์ตอ้ งระวังโครงสร้ำงใด

A.vagus

B.Recurrent laryngeal

You might also like